SEARCH (Type the topic here)

Endocrine Surgery MCQ

1. When progressive enlargement of a multinodular goiter causes symptomatic tracheal compression, the preferred management in otherwise good-risk patients is:
A. Iodine treatment.
B. Thyroid hormone treatment.
C. Surgical resection of the abnormal thyroid.
D. Radioactive iodine treatment.
Answer: C



DISCUSSION: When a multinodular goiter enlarges enough to cause symptoms of tracheal compression, surgical treatment is usually required if the patient is considered a reasonable operative risk. Medical treatment may be effective in preventing the initial growth of the goiter but is unlikely to cause enough regression to relieve symptoms. Radioactive iodine can occasionally be used to cause some regression in patients who are poor anesthesia risks, but this is a temporizing treatment rather than a definitive one.


2. The most precise diagnostic screening procedure for differentiating benign thyroid nodules from malignant ones is:
A. Thyroid ultrasonography.
B. Thyroid scintiscan.
C. Fine-needle-aspiration biopsy (FNAB).
D. Thyroid hormone suppression.
Answer: C

DISCUSSION: Analysis of multiple series in which patients with thyroid nodules have undergone FNAB has demonstrated a false-negative rate of 2.4% and a false-positive rate of 3.3%. Sensitivity for this method is 92%; specificity 74%. This surpasses the other methods for accurate selection of patients who require surgical resection.


3. The preferred operation for initial management of a thyroid nodule that is considered suspicious for malignancy by FNAB is:
A. Excision.
B. Partial lobectomy.
C. Total lobectomy and isthmusectomy.
D. Total thyroidectomy.
Answer: C

DISCUSSION: There is a consensus that the initial minimum operation for a nodule suspected to be malignant is total lobectomy and isthmusectomy. Partial lobectomy or excision of the nodule is associated with a higher risk of local recurrence if the nodule proves to be malignant. Reoperation on the side of a partial lobectomy can be technically difficult and associated with a higher risk of recurrent nerve injury. Ordinarily, total thyroidectomy is not performed until a conclusive diagnosis of malignancy is established.


4. Advantages of total thyroidectomy for management of papillary carcinomas of the thyroid larger than 1.5 cm. include:
A. Possibility of using radioactive iodine postoperatively to identify and treat metastases.
B. The ability to use thyroglobulin levels as a marker for recurrence.
C. Lower overall recurrence rate.
D. Lower risk of hypoparathyroidism.
Answer: ABC

DISCUSSION: Following total thyroidectomy iodine 131 can be used more efficiently because of the absence of normal thyroid tissue, which has greater affinity for iodine than papillary carcinoma tissue. When all normal thyroid tissue is removed, serum thyroglobulin, which is produced by normal and malignant thyroid tissue, becomes a more effective marker for recurrence. The overall recurrence rate is lower for patients undergoing total thyroidectomy, but the risk of hypoparathyroidism is higher for patients who have total thyroidectomy instead of unilateral lobectomy.


5. Which of the following statements about follicular carcinoma is/are true?
A. It presents at a later age than papillary carcinoma.
B. It disseminates via hematogenous routes.
C. It is the most common type of well-differentiated thyroid carcinoma.
D. Extensive angioinvasion portends a poor prognosis.
E. Follicular carcinomas are frequently multicentric.
Answer: ABD

DISCUSSION: Follicular carcinoma is more common in older patients (peak incidence in the fifth decade). The tumor has a marked propensity for vascular invasion and spreads hematogenously to bone, lung, liver, and central nervous system sites. Local nodal metastases are less common than in papillary carcinoma. Extensive angioinvasion indicates a less favorable prognosis. Papillary carcinoma is the most common type of well-differentiated thyroid carcinomas. Follicular carcinomas are rarely multicentric.


6. A familial form of medullary thyroid carcinoma (MTC) should be suspected whenever:
A. The tumor is multifocal.
B. The tumor is bilateral (foci of tumor are present in both thyroid lobes).
C. Pathologic examination of the resected thyroid gland reveals the presence of C-cell hyperplasia in areas of the gland adjacent to foci of MTC.
D. All of the above.
Answer: D

DISCUSSION: Sporadic MTC is unilateral in at least 80% of cases. However, in patients with MTC occurring as a component of the multiple endocrine neoplasia (MEN) type 2A or type 2B syndromes, the tumor is virtually always multifocal and bilateral. Typically, in this setting the MTC appears as multiple whitish-tan tumor nodules in the middle and upper thirds of each thyroid lobe.
A diffuse premalignant proliferation of the C cells of the thyroid is thought to precede the development of MTC in patients with familial MTC. This proliferation, known as C-cell hyperplasia (CCH), consists of parafollicular clusters of increased numbers of C cells. The finding of CCH in areas of the thyroid adjacent to gross foci of MTC is strong evidence for familial MTC.


7. All of the following are components of the MEN type 2B syndrome except:
A. Multiple neuromas on the lips, tongue, and oral mucosa.
B. Hyperparathyroidism.
C. MTC.
D. Pheochromocytoma.
Answer: B

DISCUSSION: MTC and pheochromocytoma occur in both MEN 2A and MEN 2B syndromes. Patients with MEN 2A may also develop hyperplasia of the parathyroid glands. Although some investigators have reported equivocal histologic abnormalities in the parathyroid glands of patients with MEN 2B, hyperparathyroidism is not a component of this syndrome.
In contrast to patients with MEN 2A, those with MEN 2B have a characteristic phenotype, including a tall, thin “marfanoid” habitus. Patients with MEN 2B also develop multiple neuromas on the lips, tongue, and oral mucosa, creating the appearance of thick lips.


8. MEN 2A and MEN 2B syndromes are associated with germline mutations in:
A. The p53 tumor suppressor gene.
B. The H-ras gene.
C. The N-myc gene.
D. The RET proto-oncogene.
Answer: D

DISCUSSION: Germline mutations in the RET protooncogene, a receptor tyrosine kinase that maps to chromosome 10, are associated with MEN 2A and MEN 2B syndromes. Homozygous loss of the tumor suppressor gene p53 is associated with the Li-Fraumeni syndrome, and mutations of p53 are present in a variety of human neoplasms. Point mutations in the H-ras gene are associated with carcinoma of the colon, lung, and pancreas. Amplification of the N-myc gene, when present in neuroblastoma, suggests a poorer prognosis.


9. Which of the following are true concerning islet cell neoplasms of the pancreas in patients with MEN type 1?
A. Islet cell neoplasms in patients with MEN 1 are characteristically multicentric.
B. The most common islet cell neoplasm in patients with MEN 1 is gastrinoma.
C. Islet cell neoplasms in patients with MEN 1 may be malignant.
D. All of the above.
Answer: D

DISCUSSION: The pathologic change in the pancreas of patients with MEN 1 is typically multicentric. Diffuse hyperplasia of islet cells and microadenoma formation are often identified in areas of the gland distant from grossly evident tumor. Tumors are commonly multifocal. Islet cell neoplasms of the pancreas occur in 30% to 80% of patients with MEN 1. The most common islet cell neoplasm in these patients is gastrinoma. Gastrinomas associated with MEN 1 probably account for 20% to 50% of all cases of the Zollinger-Ellison syndrome. The second most common islet cell tumor is insulinoma. Other pancreatic islet cell neoplasms, such as glucagonoma, somatostatinoma, or vasoactive intestinal polypeptide neoplasm (VIPoma), are rarely associated with MEN 1. Approximately 10% of insulinomas and approximately 15% or more of gastrinomas in patients with MEN 1 are malignant.


10. Which of the following statements about the differential diagnosis of hypercalcemia is/are correct.
A. Malignant tumors typically cause hypercalcemia by ectopic production of parathyroid hormone (PTH).
B. The diagnosis of primary hyperparathyroidism is supported by these serum levels: calcium, 10.8 mg. per dl.; chloride, 104 mmol. per liter; bicarbonate 21 mmol. per liter; phosphorus, 2.4 mg. per dl.; elevated parathyroid hormone.
C. Familial hypocalciuric hypercalcemia is distinguished from primary hyperparathyroidism by parathyroid imaging.
D. Although serum albumin binds calcium, the measured total calcium value is usually unaffected in patients with severe hypoproteinemia.
E. Thiazide diuretics are a good treatment for hypercalcemia and can be given to patients with apparent hypercalcemia of malignancy.
Answer: B

DISCUSSION: Malignant tumors rarely secrete PTH itself; they can secrete PTHrP or cytokine activators of osteoclast activity. The diagnosis of primary hyperparathyroidism is supported by hypercalcemia with mild hyperchloremic metabolic acidosis and a chloride-phosphate ratio greater than 33 or a modified chloride (mmol. per liter/mg. per dl.) phosphate ratio greater than 500. Familial hypocalciuric hypercalcemia is distinguished from primary hyperparathyroidism by a low urine calcium. Serum calcium changes approximately 0.8 mg. per dl. for every 1 gm. per dl. change in serum albumin. Thiazide diuretics can cause hypercalcemia and should not be given to patients who are hypercalcemic.


11. Indications for operation in a patient with previously asymptomatic hyperparathyroidism include:
A. Age older than 60.
B. Nephrolithiasis.
C. A substantial decline in renal function.
D. A substantial decline in bone mass.
E. Depression and fatigue.
Answer: BCDE

DISCUSSION: Age younger than 50 is considered an indication for operation.


12. The parathyroid glands:
A. Develop from the second and third pharyngeal pouches, along with the palatine tonsil and the thymus.
B. Migrate caudally in the neck in normal development but can be found anywhere from the pharyngeal mucosa to the deep mediastinum.
C. Secrete PTH and calcitonin to manage calcium homeostasis.
D. Usually number four, but frequently number only two or three.
E. Contain enzymes that catalyze the conversion of 25(OH) vitamin D 3 to 1,25(OH) 2 vitamin D 3.
Answer: B

DISCUSSION: The parathyroid glands develop from the third and fourth pharyngeal pouches, along with the thymus and the thyroid ultimobronchial body; there are four glands in the vast majority of persons. Calcitonin is secreted by the C cells of the thyroid. Vitamin D 3 hydroxylation occurs in the kidney.


13. Hyperparathyroidism can affect which of the following organs and body systems?
A. Gastrointestinal tract.
B. Kidneys.
C. Skeleton.
D. Neuromuscular system.
E. Cardiovascular system.
Answer: ABCDE

DISCUSSION: Gastrointestinal involvement includes pancreatitis and peptic ulcer disease. The kidneys can be affected by nephrocalcinosis or nephrolithiasis. The bones can be severely affected by bone resorption. Neuromuscular complaints can include weakness and fatigue. Hypertension is present in as many as 70% of people with hyperparathyroidism and may be related to the renal disease.


14. Secondary hyperparathyroidism:
A. Is a metabolic disease in which the primary abnormality is decreased glomerular filtration rate.
B. Is best treated initially by subtotal parathyroidectomy.
C. Is caused by increased production of 1,25(OH) 2 vitamin D 3, causing increasing intestinal calcium absorption and hypercalcemia.
D. Can have severe effects on bones exacerbated by aluminum contained in phosphate binders and dialysate water.
E. Is best treated initially by total parathyroidectomy with autotransplantation.
Answer: AD

DISCUSSION: Secondary hyperparathyroidism is caused by renal disease, including decreased glomerular filtration rate and decreased 1,25(OH) 2 vitamin D 3 production. It is best treated by medical management restricting dietary phosphate, administering vitamin D and calcium with phosphate-binding gels, and limiting aluminum exposure, which can exacerbate bone disease.


15. Hypoparathyroidism:
A. Is most commonly encountered as a postviral syndrome.
B. Can be associated with marked hypocalcemia after parathyroidectomy in patients with bone disease.
C. Can cause anxiety, depression, or confusion.
D. Can cause physical signs such as Chvostek's and Trousseau's signs.
E. Is treatable acutely with intravenous calcium salts and chronically with oral calcium and vitamin D.
Answer: BCDE

DISCUSSION: Hypoparathyroidism is most commonly encountered after thyroid surgery. It can be temporarily severe after parathyroidectomy in patients with bone disease (“postoperative bone hunger”). The signs and symptoms can include anxiety, depression, confusion, Chvostek's sign, and Trousseau's sign, as well as circumoral or extremity tingling, tetany with carpopedal spasms, or seizures. Treatment is as noted.


16. True statements about pituitary anatomy and physiology include:
A. The pituitary has dual embryonic origin: the anterior pituitary arises from embryonic ectoderm; the posterior pituitary, from the diencephalon.
B. The hypophyseal portal system integrates function of the anterior and posterior pituitary.
C. Adrenocorticotropin (ACTH), formed by posttranslational processing of the precursor POMC, is normally controlled by hypothalamic CRF but may be released by immune-related mechanisms.
D. Growth hormone (GH) directly stimulates longitudinal growth of the skeleton and growth of muscles.
E. Cell types of the anterior pituitary are classified by their position in the anterior pituitary and by their staining characteristics with histologic dyes.
Answer: AC

DISCUSSION: The anterior pituitary arises from Rathke's pouch (embryonic ectoderm), and it includes the pars distalis, pars intermedia, and pars tuberalis. The posterior pituitary arises from the diencephalon and includes the neural stalk, infundibulum, and posterior lobe. The hypophyseal portal system drains from the hypothalamus and integrates function of the hypothalamus and anterior pituitary by carrying hypothalamus-derived releasing factors to target cells in the anterior pituitary. ACTH may be released during stress in response to interleukin-1, -2, and -6. Somatotrophic actions of GH are indirect and are mediated by the insulinlike growth factors formerly known as somatomedins. Current classification of anterior pituitary cells is based on immunochemical identification of their secretory products (i.e., corticotropes produce ACTH, lactotropes produce prolactin, and thyrotropes produce thyroid-stimulating hormone [TSH]).


17. Antidiuretic hormone (ADH):
A. Is related to oxytocin, and both are released from the posterior pituitary in conjunction with neurophysins.
B. Is released into the circulation by the posterior pituitary in response to a rise in plasma osmolality above 285 mOsm. or a decrease in circulating blood volume.
C. May be stimulated by catecholamines and inhibited by phenytoin, alcohol, and lithium.
D. In excess, may produce a syndrome of euvolemic hyponatremia with inappropriately concentrated urine that is responsive to free water restriction.
E. Deficiency causes prolonged polyuria and polydipsia and may be diagnosed by a combination of high plasma osmolality and low urine osmolality following water deprivation.
Answer: ABCDE

DISCUSSION: ADH and oxytocin are nine–amino acid peptides derived from a common ancestral peptide, vasotocin. ADH is released from the posterior pituitary with neurophysin II in response to a rise in plasma osmolality greater than 285 mOsm/kg. H 2O. by a 5% or greater decrease in blood volume, and by catecholamines. The syndrome of inappropriate ADH release (SIADH) produces euvolemic hyponatremia and is responsive to free water restriction. Diabetes insipidus (DI) reflects a deficiency of ADH and causes prolonged polyuria and polydipsia. Diabetes insipidus is diagnosed by a combination of high plasma osmolality and low urine osmolality following water deprivation. Exogenously administered ADH differentiates central (ADH-responsive) DI from nephrogenic (ADH-unresponsive) DI.


18. Signs and symptoms of acute pituitary apoplexy include:
A. Severe headache.
B. Meningismus.
C. Vision loss.
D. Shock.
E. May be relieved by emergent transsphenoidal decompression of the sella turcica.
Answer: ABCD

DISCUSSION: Acute pituitary apoplexy follows sudden hemorrhage into a pre-existing pituitary tumor or following closed head trauma. Symptoms, including headache, meningismus, and vision loss, are attributable to the intracerebral blood. Pituitary insufficiency, as well as the accompanying secondary adrenal insufficiency, may cause hypotension and shock. Other manifestations may include DI and myxedema. Acute pituitary apoplexy is a neurosurgical emergency that requires transsphenoidal decompression of the sella turcica.


19. Prolactinomas of the pituitary:
A. Most often produce dysfunctional uterine bleeding in women.
B. Most commonly produce infertility in men.
C. When asymptomatic, are best treated surgically early in the microadenoma stage.
D. May enlarge during pregnancy, requiring treatment with bromocriptine or surgery.
E. Commonly occur in patients with MEN 2.
Answer: D

DISCUSSION: Pituitary prolactinomas are the most common pituitary tumor and cause amenorrhea, galactorrhea, or both in women. Men usually have macroprolactinomas and experience symptoms of a space-occupying lesion of the sella (i.e., headache). A minority of men experience impotence and infertility. Asymptomatic prolactinomas remain stable over time and require observation only. Symptomatic prolactinomas require treatment, usually with bromocriptine. Prolactinomas may enlarge during pregnancy and can produce symptoms requiring treatment with either bromocriptine or surgery. Prolactiomas are seen in as many as 40% of patients with MEN 1.


20. Hypercortisolism:
A. Is most often ACTH-dependent, owing to an ACTH-producing pituitary adenoma.
B. Is best diagnosed by measurement of cortisol from a serum sample collected at 8 A.M.
C. Is attributable to an adrenal source if the basal serum ACTH level is above 10 pg. per ml., if the hypercortisolism is suppressed by high-dose dexamethasone, and if an adrenal tumor is visualized radiographically.
D. May be caused by small cell carcinoma of the lung, carcinoid tumors, tumors of the endocrine pancreas, pheochromocytoma, or medullary thyroid carcinoma (MTC).
E. In children is most often caused by adrenocortical neoplasia.
Answer: ADE

DISCUSSION: Cushing's syndrome is most often (80% to 90%) ACTH-dependent and is most often due to an ACTH-secreting pituitary adenoma (Cushing's disease). Some 10% to 20% of ACTH-dependent Cushing's syndrome is caused by ectopic production of ACTH from small cell carcinoma of the lung, carcinoid tumors, tumors of the endocrine pancreas, pheochromocytoma, and MTC. ACTH-independent Cushing's syndrome secondary to primary adrenal pathology occurs in 10% to 20% of cases in adults and is the most common form in children. Measurement of cortisol in two to three consecutive 24-hour collections of urine is the best screening test for Cushing's syndrome: plasma levels of cortisol show marked variability, and a single random level is not helpful in establishing the diagnosis. ACTH-independent Cushing's syndrome suppresses the pituitary and is diagnosed if the basal serum ACTH level is suppressed below 5 pg. per ml., if the hypercortisolism is not suppressed by high-dose dexamethasone, or if the metyrapone stimulation test is negative. The dexamethasone suppression and metyrapone tests do not, however, distinguish an adrenal from an ectopic ACTH cause of Cushing's since both suppress the pituitary. Radiographic evidence of an adrenal tumor must be supported by biochemical testing to confirm that the adrenal is the primary cause of Cushing's syndrome.


21. Primary aldosteronism:
A. Produces a syndrome of diastolic hypertension, hypokalemia, and edema.
B. Is suggested by findings of serum potassium less than 3.5 mEq. per liter, urinary potassium excretion greater than 30 mEq. per day, upright plasma renin below 3 ng. per ml., and a plasma aldosterone concentration–plasma renin activity ratio greater than 20:1.
C. Is most often due to an aldosterone-producing adrenal adenoma, which may be distinguished from idiopathic adrenal hyperplasia by its sensitivity to diurnal changes in ACTH and insensitivity to changes in posture.
D. May be diagnosed in hypertensive patients by demonstration of an adrenal mass larger than 1 cm. on computed tomography alone.
E. Is best treated surgically if it is due to either aldosteronoma or idiopathic adrenal hyperplasia.
Answer: BC

DISCUSSION: Primary aldosteronism is a syndrome of diastolic hypertension and hypokalemia; edema typically is absent. The diagnosis relies on demonstration of hypokalemia with inappropriate kaliuresis and inappropriately elevated plasma aldosterone with suppression of plasma renin following sodium loading. Primary aldosteronism most often is due to an adrenal adenoma, which must be distinguished from idiopathic adrenal hyperplasia since resection of the adenoma is nearly always curative, whereas adrenal resection for idiopathic adrenal hyperplasia is curative less than 20% of the time. CT evidence of adrenal tumor alone is inadequate to diagnose aldosteronism, even in a hypertensive patient.


22. Adrenocortical carcinoma:
A. May be suspected in a patient with rapidly progressive Cushing's syndrome and virilizing features or in asymptomatic patients with adrenal tumors larger than 6 cm on CT.
B. Most often is diagnosed early in its course when disease is confined to the adrenal gland.
C. Is differentiated from benign adrenocortical adenoma by tumor necrosis, hemorrhage, and cellular features of large hyperchromatic nuclei and more than 20 mitoses per high-power field.
D. Should be resected only if disease is localized to the adrenal gland; otherwise treatment with mitotane is indicated.
E. Carries a poor prognosis: overall 5-year survival less than 25%.
Answer: AE

DISCUSSION: Patients with adrenocortical carcinoma often present with rapidly progressive syndromes of combined adrenocortical hormone excess. However, approximately half do not have syndromes of hormone excess but present with abdominal pain, increased abdominal girth, weight loss, and anorexia. Furthermore, incidentally discovered adrenal masses larger than 6 cm. harbor carcinoma in as many as 92% of cases. Adrenocortical carcinoma is differentiated from adenoma by pathologic demonstration of either local invasion or distant metastases. Most patients with adrenocortical carcinoma present with locally advanced (stage III) or metastatic (stage IV) disease. Prognosis for these patients is poor: 5-year survival is less than 25% in most series. Patients with adrenocortical carcinoma should undergo primary surgical resection or palliative surgical debulking of locally advanced, metastatic, or even recurrent disease since response of these tumors to medical therapy, including mitotane, is poor.


23. Addisonian crisis, or acute adrenocortical insufficiency:
A. Occurs only in patients with known adrenal insufficiency or in those receiving long-term supraphysiologic doses of exogenous steroids.
B. Can mimic an acute abdomen with fever, nausea and vomiting, abdominal pain, and hypotension.
C. May cause electrolyte abnormalities, including hypernatremia, hypokalemia, hypoglycemia, and hypercalcemia, as well as eosinophilia on peripheral blood smear.
D. Should be diagnosed with the rapid ACTH stimulation test before steroid replacement is instituted.
E. May be effectively treated with intravenous “stress-dose” glucocorticoid and mineralocorticoid replacement.
Answer: B

DISCUSSION: Addisonian crisis, or acute adrenal insufficiency, may be seen following even mild illness in patients with suppression of the hypothalamic-pituitary-adrenal axis. This suppression can be produced by as little as 1 week of supraphysiologic stress dose steroids in the year before the stressful event. Addisonian crisis is a medical emergency that requires prompt treatment based on clinical suspicion. Clinical findings include fever, nausea and vomiting, abdominal pain, and hypotension. Laboratory analysis may reveal electrolyte abnormalities, including hyponatremia, hyperkalemia, hypoglycemia, and hypercalcemia, as well as eosinophilia on peripheral blood smear. The rapid ACTH test is diagnostic, but it should not delay treatment with intravenous fluid resuscitation, glucose replacement, and high-dose dexamethasone. Dexamethasone, not hydrocortisone, should be given initially, since it does not interfere with subsequent determination of plasma cortisol. Stress dose steroids are inadequate once adrenal crisis has occurred, and exogenous mineralocorticoids are given when the patient resumes oral intake.


24. Preparation for surgical removal of a pheochromocytoma includes:
A. Beta-adrenergic blockade followed by alpha-adrenergic blockade.
B. Hydration.
C. Alpha-adrenergic blockade, with or without beta-adrenergic blockade.
D. Preoperative Swan-Ganz monitoring in all patients.
E. Planning removal through an anterior, posterior, or laparoscopic approach based upon tumor localization with CT, magnetic resonance imaging (MRI), and/or 131I-MIBG.
Answer: BCE

DISCUSSION: Principles of preoperative management for pheochromocytoma include preoperative alpha-adrenergic blockade using phenoxybenzamine or phentolamine. Beta-adrenergic blockade with propranolol is then used selectively in patients who develop tachycardia, have a history of cardiac arrhythmia, or have primarily epinephrine-secreting tumors. Beta-adrenergic blockade should be undertaken only after successful alpha blockade is established. Patients with pheochromocytoma frequently exhibit intravascular volume depletion, and careful hydration is mandatory. Central venous pressure monitoring alone is helpful to guide hydration; more intensive monitoring with a Swan-Ganz catheter is indicated for patients with pre-existing heart disease. Formerly, the anterior approach was preferred for adrenalectomy as it facilitated complete abdominal exploration and search for extra-adrenal pheochromocytoma. Accurate preoperative localization with CT, MRI, and 131I-MIBG has allowed selective use of the posterior, or even the laparoscopic, approach for adrenalectomy.


25. Indications for surgical adrenalectomy include:
A. An adrenal mass larger than 6 cm.
B. Hypertensive patients with aldosteronism that is ACTH insensitive and posture sensitive and who also have multiple adrenal nodules on CT.
C. Cushing's syndrome secondary to adrenal neoplasms or to persistent ectopic ACTH syndrome when the primary tumor is inoperable.
D. Pheochromocytoma in adults and children.
E. Congenital adrenal hyperplasia secondary to 21-hydroxylase deficiency.
Answer: ACD

DISCUSSION: Adrenalectomy is indicated for the management of adrenal masses larger than 6 cm. on CT since tumors of this size harbor carcinoma in more than 90% of cases. Surgical adrenalectomy is also the treatment of choice for primary adrenal causes of Cushing's syndrome. Surgical adrenalectomy may be considered as an alternative to medical adrenalectomy with metyrapone, aminoglutethimide, or mitotane in patients with ectopic ACTH syndrome when treatment of the primary tumor is unsuccessful. Patients with aldosteronism that is ACTH sensitive and posture insensitive likely have an adrenal adenoma that is curable by adrenalectomy. Aldosteronism that is ACTH insensitive and posture sensitive is likely caused by idiopathic adrenal hyperplasia, which is best managed medically with spironolactone, triamterene, amiloride, or nifedipine. Adrenalectomy following preoperative alpha-adrenergic blockade, with or without beta-adrenergic blockade, and hydration is the treatment of choice of all pheochromocytomas. Adrenalectomy is not indicated in the management of any of the congenital adrenal hyperplasias.


26. Incidental adrenal masses:
A. May be seen in as many as 10% of abdominal CT studies.
B. Most commonly represent pheochromocytoma; adrenocortical adenoma, adrenocortical carcinoma, and metastases from other primary cancers occur less frequently.
C. May represent adrenocortical carcinoma if greater than 6 cm. in diameter.
D. Should be routinely evaluated by measurement of 24-hour urine levels of catecholamines and their metabolites, cortisol, and aldosterone plus fine-needle aspiration.
E. Should be resected if biochemically active, if greater than 6 cm., or if they grow over six months' follow-up.
Answer: CE

DISCUSSION: The incidental adrenal mass is seen in as many as 1.3% of abdominal CT scans performed for other reasons. Adrenocortical adenomas are most common, followed by adrenocortical carcinoma, metastases from other primary cancers, and pheochromocytoma. Biochemical evaluation must weigh the prevalence of adrenal neoplasms against the consequences of a missed life-threatening diagnosis, as in pheochromocytoma. All adrenal masses should be evaluated for pheochromocytoma with measurement of 24-hour urine catecholamines and their metabolites. Aldosterone and cortisol measurement are indicated if clinical features suggest aldosteronism or Cushing's syndrome. Fine-needle aspiration of adrenal masses is indicated for clearly cystic lesions or if metastasis is suspected based on the presence of another known primary. Fine-needle aspiration is not routinely indicated in the evaluation of adrenal lesions and is contraindicated until pheochromocytoma is definitively excluded. Adrenal lesions should be resected if they are functional, are larger than 6 cm., or have enlarged during follow-up.


27. Which of the following statements is true about the synthesis of thyroid hormone and its physiology?
A. The iodine utilized in hormone synthesis is derived principally from dietary sources.
B. The role of thyroid-stimulating hormone (TSH) in thyroid physiology is limited to regulation of the release of thyroid hormone in plasma.
C. Enough thyroxine (T 4) is stored in the normal thyroid to provide a euthyroid state for 3 weeks despite absence of iodine intake.
D. The regulation of thyroid function involves pituitary, but not hypothalamic, input.
Answer: AC

DISCUSSION: Iodine is necessary for the synthesis of thyroid hormone, and approximately 200 to 500 mg is ingested daily. Most of it is absorbed from the small intestine and is cleared from the plasma by secretion of thyroid hormone. TSH is required for the normal production and secretion of thyroid hormone. It is generally accepted that TSH also has a major role in thyroid growth. The thyroid gland has a storage reserve of approximately 3 weeks. The principal regulatory mechanisms of the thyroid gland are the hypothalamic-pituitary-thyroid control system and the intrathyroidal regulatory systems. TRH is produced by the superoptic and paraventricular nuclei within the hypothalamus and passes down their axones. Following secretion into the hypophyseal portal blood systems TRH passes to the pituitary and induces stimulation of TSH secretion.


28. Correct statements about thyroid function tests include which of the following?
A. Contraceptive pills and pregnancy increase the amount of thyroxin-binding globulin (TBG), and, consequently, the total T 4 level.
B. Anticonvulsive medications and chronic debilitating illnesses decrease the amount of TBG and, consequently, the total T 4 level.
C. Intravenous pyleography can lower the rate of active iodine uptake by the thyroid.
D. A triiodothyronine (T 3) suppression test that demonstrates nonsuppressibility of thyroid function is compatible with the diagnosis of Graves' disease, toxic adenoma, or functioning carcinoma.
E. An increased serum cholesterol level in a hypothyroid patient indicates a thyroid cause.
Answer: ABCD

DISCUSSION: Hormone binding proteins are the principal intravascular factors influencing total hormone concentration. Various factors may cause changes in the concentration of TBG. Contraceptive pills and pregnancy increase the amount of TBG, while anticonvulsive medications and chronic debilitating disease may decrease the amount of TBG. Administration of excess amounts of iodine may lower the rate of active iodine uptake in the thyroid gland. The thyroid suppression test is based on the principle that administration of thyroid hormone does not suppress the patient's thyroid function when normal homeostatic mechanisms are disrupted. Thyroid function is nonsuppressible in hyperthyroidism or in the presence of thyroid hormone–secreting tumors. Serum cholesterol, deep tendon reflex time, and cardiovascular functions can be measured to reflect hypothyroidism and hyperthyroidism but do not give an indication of the etiology of hypothyroidism.


29. Hyperthyroidism can be caused by all of the following except:
A. Graves' disease.
B. Plummer's disease.
C. Struma ovarii.
D. Hashimoto's disease.
E. Medullary carcinoma of the thyroid.
Answer: E

DISCUSSION: Common types of hyperthyroidism include diffuse toxic goiter (Graves' disease, named after the Dublin physician Robert Graves who described it in 1835 but known since its original description by Parry in 1786 and described by von Basedow in 1840) and toxic adenoma or toxic multinodular goiter (Plummer's disease). Uncommon causes include thyrotoxicosis factitia, functioning metastatic thyroid carcinoma, trophoblastic tumors that secrete human chorionic gonadotropin (having thyroid-stimulating properties), inappropriate secretion of thyrotropin by pituitary tumors, struma ovarii, iodide-induced hyperfunction, and thyroiditis.


30. Which of the following is true about the use of radioiodine to treat hyperthyroidism?
A. If hyperthyroidism is secondary to radioiodine use, it will occur within 2 years of treatment.
B. There is a markedly increased risk of future thyroid cancer following radioiodine therapy.
C. The risk of leukemia following radioiodine therapy is approximately 10%.
D. Mutation abnormalities occur in 15% of fetuses in utero following internal treatment of the mother with radioiodine during pregnancy.
E. Radioiodine may pass through the placenta and lactating breast to produce hypothyroidism in a fetus or infant.
Answer: E

DISCUSSION: Hypothyroidism secondary to radioiodine therapy increases in frequency with time to 85% after 5 years. There has been no demonstration of an increase in malignancies following radioiodine therapy. Radioactive iodine can cross the placenta and lactating breast to produce hypothyroidism in a fetus or a nursing infant and is, therefore, contraindicated.


31. Arrange the following complications of thyroid surgery (bilateral subtotal thyroidectomy) in decreasing order of incidence in patients with Graves' disease.
A. Laryngeal nerve paralysis.
B. Hypoparathyroidism.
C. Hypothyroidism.
D. Recurrent hyperthyroidism.
Answer: CDAD

DISCUSSION: The incidence of recurrent disease is inversely related to the incidence of hypothyroidism and is 1% to 5%. Within 1 to 2 years, hypothyroidism may develop in 5% to 50% of patients (with a slight additional increase in subsequent years). The associated morbidity—related primarily to damage to the recurrent laryngeal nerves and parathyroid glands—is estimated to be 0.5% to 3.0%.


32. The most common cause of goitrous hypothyroidism in adults is:
A. Graves' disease.
B. Riedel's thyroiditis.
C. Hashimoto's disease.
D. de Quervain's thyroiditis.
Answer: C

DISCUSSION: Hashimoto's disease was first described in Japan by Hakaru Hashimoto in 1912 and is the best-known of the immunologic thyroid diseases. It is the most common cause of goitrous hypothyroidism in adults and of sporadic goiter in children. The incidence is 0.3 to 1.5 cases per 1000 population per year and it is 10 to 15 times more common in women than in men, with the highest incidence in the group aged 30 to 50 years.


33. Therapy for Hashimoto's disease includes:
A. Radioiodine.
B. Antithyroid medications.
C. Subtotal thyroidectomy.
D. None of the above.
Answer: D

DISCUSSION: There is no specific treatment for Hashimoto's disease. Patients are usually followed medically, and replacement therapy with T 4 is begun in patients with hypothyroidism that is symptomatic or associated with a goiter that is causing pressure symptoms. Early initiation of thyroid hormone therapy has been recommended by many to prevent further thyroid enlargement and reduce the risk of myxedema, especially in postpartum patients. Surgical reduction of goiter should be performed if severe pressure symptoms that have not responded to corticosteroid therapy are present. This usually consists of subtotal thyroidectomy. Biopsy to rule out malignancy in nodules suspicious for thyroid carcinoma (usually papillary) or lymphoma is indicated. If carcinoma is suspected, lobectomy should be performed, and if frozen section demonstrates carcinoma, subtotal or total thyroidectomy should be performed.


34. Indications for surgical thyroidectomy for Graves’ disease include which of the following?

a. Ocular involvement
b. Symptomatic large goiter
c. Women of childbearing age
d. Concomitant thyroid nodule
e. All of the above
Answer: b, c, d

Antithyroid drugs are the initial therapy in most patients with Graves’ disease, either as a definitive therapy or in preparation for 131I therapy or surgical ablation. Because of the high failure rate of long-term treatment with thionamides, the use of these drugs as definitive treatment has decreased. Drawbacks include the important 0.5% incidence of agranulocytosis and a recurrence rate as high as 43% during the first year when the drug is stopped. Five years after treatment only 25% of patients remain in remission. Unlike other definitive treatments, hypothyroidism does not occur as a result of thionamide treatment if an appropriate dosage is used. 131I has been used as definitive treatment for patients with Graves’ disease for many years with predictable and long-lasting good results in most patients. It has few, if any, serious side effects. It is ablative to the thyroid gland and hypothyroidism is a nearly inevitable result of effective therapy, although it may take years to become clinically apparent. About 70% of patients treated with 131I are hypothyroid within 10 years of treatment. The risk of recurrence of hyperthyroidism after an initial response is less than 5%. Most adult patients in the United States are treated with 131I as definitive treatment for Graves’ disease. Exceptions are women in the childbearing years where a subsequent fetus would be affected, patients with concomitant thyroid nodules where carcinoma is a concern, those with extremely large glands and, increasingly, those who are opposed to 131I therapy. Thyroidectomy is an important alternative in selected patients with Graves’ disease. Although controversial in the past, it now appears that ocular involvement does not respond more favorably to thyroidectomy than to 131I ablation.


35. Which of the following statements regarding anatomic relationships of the thyroid gland are true?

a. The middle thyroid artery is intimately related to the superior laryngeal nerve
b. The superior thyroid artery is usually the first branch of the external carotid artery
c. Thyroidea ima arteries are found in approximately 20% of individuals
d. The parathyroid glands may lie within the pretracheal fascia
Answer: b, d

The thyroid gland is a vascular organ supplied by four main arteries: two superior and two inferior. The superior thyroid artery usually arises as the first branch of the external carotid artery just above the bifurcation of the common carotid artery. The superior thyroid artery descends medially on the surface of the inferior pharyngeal constrictor muscle to divide into an anterior and posterior branch at the apex of the thyroid lobe on its anteromedial surface. Its relation to the external branch of the superior laryngeal nerve is important during thyroid lobectomy. The inferior thyroid arteries usually arise from the thyrocervical trunks and ascend behind the carotid sheath before passing downward and medial to enter the thyroid gland at its middle portion. There are no arteries directly entering the lower poles from below with the exception of a thryoidea ima artery that may replace an absent inferior artery. Thyroidea ima arteries arise from either the innominate artery or aorta in 1% to 4% of individuals, entering the lower surface of the isthmus after coursing on the trachea.
The pretracheal fascia is referred to as the thyroid sheath and it varies in consistency and completeness among individuals. Posteromedially, it is usually condensed and firmly attaches the thyroid gland to the upper two or three tracheal rings and the cricoid cartilage. The superior parathyroid gland may lie between the sheath and the thyroid capsule, within the sheath, posterior to it in a potentially open plane, or occasionally anterior to the sheath. The inferior parathyroid gland likewise may be within the sheath, particularly when the gland is adherent to the lower pole of the thyroid.


36. Radioactive iodine is effective treatment for metastatic lung disease for which of the following thyroid neoplasms?

a. Hürthle cell carcinoma
b. Papillary carcinoma
c. Follicular carcinoma
d. Medullary carcinoma
e. Anaplastic carcinoma
Answer: b, c

Radioactive iodine is used only in patients who have differentiated thyroid carcinomas. It is of no value in the treatment and follow-up of patients with Hürthle cell, medullary or anaplastic carcinomas. Most papillary carcinomas are capable of taking up radioactive iodine. Most papillary carcinomas in patients under 50-years of age do so, providing that the patient has had a total thyroidectomy and there is no normal thyroid tissue to compete for the 131I. About 20% of all papillary carcinomas do not trap sufficient iodine for imaging or therapy. These are usually patients with papillary carcinoma variants: a tall cell variant of papillary carcinoma, insular carcinoma, or clear cell carcinoma.
Nearly all metastatic follicular carcinomas retain the ability to trap 131I sufficiently for imaging and for therapy. Even well-differentiated papillary and follicular carcinoma cannot compete successfully for 131I with normal thyroid tissue and unless this has been removed or subsequently ablated with an initial dose of 131I, many metastases cannot be detected or treated.


37. The types of thyroiditis that can cause abnormalities of surgical significance are which of the following?

a. Chronic lymphocytic thyroiditis (Hashimoto disease)
b. Riedel struma
c. Acute (viral) thyroiditis
d. Granulomatous (subacute) thyroiditis
Answer: a, b, d

There are three types of thyroiditis that can cause thyroid abnormalities of surgical significance. The most common is chronic lymphocytic (Hashimoto) thyroiditis, an autoimmune disease that can occur in any age group. Occasionally, Hashimoto disease causes unilateral thyroid enlargement that simulates malignancy. The rarest form of thyroiditis is Riedel struma which can mimic a diffuse thyroid carcinoma because of the fibrotic infiltrative process that results. Hashimoto thyroiditis is associated with reduced functional capacity of the thyroid which increases TSH secretion, and a goiter develops. Because of the associated fibrosis, a nodular goiter or neoplasm is suggested. Thyroidectomy may be indicated for treatment of a solitary nodule, particularly if it is cold, suspicious, definitely malignant or solid, and fine needle aspirate is indeterminate.
Granulomatous, DeQuervain, and subacute thyroiditis are terms that refer to a disease that usually occurs in young women within weeks of an upper respiratory or other viral infection. The disease is usually self-limited, but may persist for several months longer. In unusual patients, the disease may be confined to one lobe and result in a firm, slightly tender mass suggesting carcinoma. Lobectomy may be indicated to rule out the presence of malignancy. Total thyroidectomy may be considered for persistent, painful thyroiditis after months of steroid therapy have failed.
Goiter with a woody or fibrous component involving the adjacent strap muscles and carotid sheaths is referred to as Riedel struma. It is rare and the cause is not known. It is associated with other types of fibrotic processes including retroperitoneal fibrosis, sclerosing cholangitis, and fibrosing mediastinitis. Although considered self-limited, the process may be associated with considerable morbidity as a result of localized pain and compression of adjacent structures such as the airway. Occasionally, tracheostomy is required. Airway compression may also require open biopsy resection of the isthmus with as much as the fibrosis as possible without endangering the recurrent laryngeal nerves.


38. The principal blood supply to the parathyroid glands is which of the following?

a. Superior thyroid arteries
b. Inferior thyroid arteries
c. Thyroidea ima arteries
d. Parathyroid arterial branches directly from the external carotid artery
e. Highly variable
Answer: b

The principal blood supply to both parathyroid glands is the inferior thyroid artery. Parathyroid glands invariably have a single end artery supplying them, and if the main trunk of the inferior thyroid artery is ligated during thyroidectomy, there is no collateral blood supply to maintain their viability. It is preferable to divide only the branch of the inferior thyroid artery medial to those that supply either of the parathyroid glands. This requires individual clamping of smaller vessels under the thyroid sheath as these vessels penetrate into the thyroid capsule. Ligation of the main trunk of the inferior thyroid artery was commonly used for bilateral subtotal thyroidectomy in the past. It did not routinely cause hypoparathyroidism only because enough collateral blood supply was maintained to each end artery to one or more parathyroid glands. This is to be avoided.


39. Which of the following statements regarding papillary thyroid carcinoma are true?

a. Seventy to 80% of new cases of thyroid carcinoma in the United States are of the papillary type
b. Total ipsilateral lobectomy and isthmus resection are adequate therapy for minimal thyroid carcinoma
c. Microscopic evidence of multicentric disease is present in 70% to 80% of cases
d. Nearly all patients less than 15 years of age have metastatic disease in local lymph nodes
Answer: a, b, c, d

Seventy to 80% of the 11,000 new patients with thyroid carcinoma diagnosed annually in the United States have papillary carcinoma. Papillary carcinomas of the thyroid include minimal thyroid carcinoma, intrathyroidal, and extrathyroidal (invasion through the true thyroid capsule) disease. Minimal thyroid carcinoma refers to those papillary carcinomas that are less than 1 cm in diameter and not associated with any clinically apparent lymph node metastases. In contrast to clinically significant papillary carcinomas, these are common and are found in 2% to 13% of adult thyroid glands serially sectioned after autopsy studies of individuals who have died from other causes. For tumors between 0.5 and 1 cm, a total lobectomy and isthmus resection are satisfactory treatment.
Most clinically significant papillary carcinomas are 1 to 4 cm in diameter and are contained within the thyroid capsule. Multicentricity is relatively common and can be found on gross sectioning of the thyroid gland in 20% to 30% of cases. Furthermore, after serial sectioning of the entire thyroid gland in patients with papillary carcinoma, microscopic foci are found in 70% to 80%. Local cervical lymph node metastases are found in about 30% of all patients with papillary carcinoma. The presence of lymph node metastases does not correlate as closely to the size of the tumor as it does to the age of the patient. The younger the patient, the greater the likelihood of metastatic lymph node involvement. Nearly all patients under 15 years of age have involved metastatic lymph nodes. The presence or absence of lymph node metastases in patients with intrathyroidal primary papillary carcinomas does not appear to have an appreciable effect on long-term survival if distant metastases are not present at the time of initial treatment.


40. A 30-year-old female presents for evaluation of a palpable thyroid nodule. Technetium-99m (99mTc) scan demonstrates a single cold nodule. The differential diagnosis includes which of the following?

a. Carcinoma
b. A nonfunctioning adenoma
c. A thyroid cyst
d. A colloid nodule
e. An autonomous nodule
Answer: a, b, c, d

Radioisotope scanning measures the functional activity of the thyroid gland and maps its correlation with physical findings. For routine scanning, 131I scanning has been replaced by either 123I or 99mTc. Because of the lower radiation exposure, cost and easy use, 99mTc is preferentially used. The differential diagnosis of a solitary nonfunctioning nodule includes carcinoma, colloid nodule, nonfunctioning adenoma, and cyst. The prevalence of carcinoma ranges from 5% to 20% in cold nodules and such lesions require further evaluation. If the scan detects other nonfunctioning areas in addition to the palpable nodule, the gland is at low risk for carcinoma because most patients with this finding have a multinodular goiter. A solitary, discrete area of increased activity is more often found in a young patient with an otherwise normal thyroid gland. Multiple hot spots are typically found in an older patient with a multinodular goiter.
A functioning solitary nodule that is independent of TSH is considered an autonomous nodule and can be the cause of hyperthyroidism. It is hot on scintigraphic scan. The presence of a cold nodule is insufficient information to determine a treatment plan. Although nearly all carcinomas are cold, most cold nodules are benign. Fine needle aspiration cytology is considered the most reliable means of evaluation for the diagnosis of thyroid nodules that are nonfunctional or hypofunctional by nuclide scan.


41. Which of the following pharmacologic agents can be used in the treatment of thyrotoxicosis to block the production of thyroid hormone?

a. Propylthiouracil
b. Propranolol
c. Methimazole
d. Carbimazole
e. Iodine
Answer: a, c, d, e

A number of substances interfere with normal production of thyroid hormone by blocking one of the relevant enzymatic steps. Iodine was the first effective drug to be used in the treatment of thyrotoxicosis. It can block the organification and coupling steps in thyroid hormone synthesis as well as prevent the release of thyroid hormone. Furthermore, iodine in large doses probably inhibits the ability of TSH to stimulate cyclic AMP release at the follicular cell membrane.
Commonly used antithyroid drugs are propylthiouracil (PTU), methimazole (Tapazole) and carbimazole. Carbimazole is widely used in Great Britain, only PTU and methimazole are commonly used in the United States. PTU interferes with the incorporation of iodine into the tyrosine residues of thyroglobulin, thus preventing oxidation of iodide to iodine. It also inhibits the peripheral conversion of T4 to T3. Although both PTU and methimazole are thionamides, methimazole does not have this peripheral effect, making PTU the preferred drug for patients with thyroid storm. About 3% of patients taking PTU demonstrate at least one side effect during the first 3 months of therapy whereas the prevalence with methimazole is about 7%. These range from minor skin rashes to agranulocytosis that can be irreversible.
b-adrenergic antagonists have been used in the treatment of hyperthyrodism. Propranolol is the most widely used of these drugs. However, b-blockade does not alter thyroid function per se. Rather, its effect is to provide symptomatic relief of hyperthyroidism because of interference with the action of thyroid hormones at the cellular level.


42. A 45-year-old woman has a solitary, nonfunctioning thyroid nodule and fine needle cytology is nondiagnostic. Which of the following is the initial surgical procedure of choice?

a. Total extracapsular thyroidectomy
b. Subtotal thyroid lobectomy and resection of the isthmus
c. Total extracapsular thyroid lobectomy, resection of the isthmus, and modified unilateral neck dissection
d. Total extracapsular thyroid lobectomy and resection of the isthmus
Answer: d

Total extracapsular thyroid lobectomy and isthmus resection is the procedure of choice when a decision has been made to surgically remove a thyroid nodule. The entire lobe with the isthmus is submitted for frozen-section pathologic examination if fine needle aspirate has not already resulted in a definitive diagnosis of carcinoma. In performing total lobectomy, both parathyroid glands are carefully preserved with their blood supply. This is done in the event that total thyroidectomy is necessary if either the frozen or permanent histologic sections confirm the presence of thyroid carcinoma. Total lobectomy offers the best opportunity for accurate histologic diagnosis and is associated with the lowest incidence of complications when the need for reoperation is considered. In one experience, 800 consecutive cases of total unilateral lobectomy were performed for benign or malignant nodules suspected of cancer, and no permanent recurrent laryngeal nerve palsies occurred. Primary total lobectomy is safer than a partial lobectomy followed by resection of the residual lobe after a delayed diagnosis of malignancy. Reoperation to complete a lobectomy is associated with a greater risk to both recurrent laryngeal nerve and the parathyroids on the ipsilateral side.
Although there is controversy as to whether a total lobectomy and isthmus resection or a total thyroidectomy is the best definitive operation for unilateral papillary carcinoma, a subtotal lobectomy is universally considered an inadequate operation. A definitive cancer operation can be accomplished with one procedure in 80% of the cases when a skilled thyroid pathologist is available for frozen-section interpretation.


43. Which of the following statements regarding fine needle aspiration cytology of a thyroid nodule are true?

a. It differentiates neoplastic and nonneoplastic nodules in most cases
b. It does not allow differentiation of papillary, medullary and anaplastic carcinoma
c. It cannot differentiate malignant and benign follicular or Hürthle cell neoplasms
d. It is not recommended when a patient has a history of head and neck radiation
Answer: a, c, d

When interpreted by a skilled cytologist, fine needle aspiration is highly accurate and is considered the preferred method of selecting patients with thyroid nodules for surgery. Nearly 80% of patients with thyroid nodules were spared surgical exploration as a result of such studies in one report. Because of the risk of false-negative diagnoses (10%), advocates of this technique emphasize the importance of clinical judgment in addition to the cytologic study in selecting operative candidates.
In most cases, fine needle aspiration cytology enables the pathologist to distinguish nonneoplastic from neoplastic nodules and to identify the type of malignant tumor. Papillary, medullary, and anaplastic carcinoma all have a typical cytologic appearance. Cytologic studies cannot differentiate malignant from benign follicular or Hürthle cell neoplasms. In this case, a definitive diagnosis depends on histologic examination of the entire excised tumor.
Fine needle aspiration has dramatically reduced the number of diagnostic surgical operations for benign lesions in centers where it is used extensively. In patients with thyroid nodules and a history of previous head and neck radiation, operation is generally recommended regardless of cytologic findings. In these patients, both benign and malignant lesions may develop and the chances of sampling error are considerable.


44. Hyperthyroidism results from all of the conditions noted below. Of the following which commonly require surgical management?

a. Graves’ disease
b. Struma ovarii
c. Functioning metastatic thyroid carcinoma
d. Toxic diffuse goiter
e. Single toxic thyroid nodule
Answer: a, d, e

Hyperthyroidism is associated with clinical manifestations related to an excess of thyroid hormone. There are three causes of primary concern to the surgeon. Graves’ disease, or toxic diffuse goiter, is most common, accounting for more than 80% of all patients with hyperthyroidism. The other two relevant causes of hyperthyroidism are toxic nodular goiter and a single toxic nodule. Common causes of hyperthyroidism that rarely require surgery are postpartum thyroiditis, iodine-induced hyperthyroidism, self-administered or iatrogenic hyperthyroidism, struma ovarii, functioning metastatic carcinoma and several rare forms of thyroiditis.


45. Which of the following statements regarding medullary carcinoma of the thyroid are true?

a. Approximately 75% of all cases are hereditary
b. The overall 10-year survival rate is less than 10%
c. Medullary carcinoma of the thyroid is associated with both multiple endocrine neoplasia IIa (MEN IIa) and multiple endocrine neoplasia IIb (MEN IIb) syndromes
d. Prophylactic total thyroidectomy is recommended for MEN IIa and MEN IIb patients after the age of 10 years
Answer: c

Medullary carcinoma of the thyroid (MCT) accounts for about 7% of all malignant tumors of the thyroid. It is familial in 20% to 30% of all cases and its secretion of a biologic marker (calcitonin) allows detection of its presence with the tumor is too small to palpate. MCT appears in three clinical settings. The first is a sporadic tumor, usually in patients 30 years or older. Second, MCT occurs as a component of the MEN IIa syndrome with or without adrenal medullary disease (pheochromocytoma) or hyperparathyroidism. A family history of thyroid carcinoma with or without pheochromocytoma is invariably present. MCT in the MEN IIa syndrome is always bilateral and multicentric and arises from C-cell hyperplasia. MCT usually does not develop before age 12 and is almost always clinically apparent before age 30.
Lastly, MCT is found as a component of the MEN IIb syndrome without or without bilateral adrenal medullary disease and always with the facies and autonomic nervous system dysplasia expressed as ganglioneuromatosis from the lips to the anus. These patients often have a marfanoid habitus and skeletal deformities as well. The MEN IIb syndrome occurs as a sporadic mutation, but its familial occurrence is becoming more common because patients are surviving now long enough to reproduce. MCT or its precursor, C-cell hyperplasia, develops by age 2 in MEN IIb and is always bilateral and multicentric. As a result of its early appearance and late detection, the disease has usually been more advanced when treatment has been instituted. Minimal treatment of MCT is total thyroidectomy. The overall prognosis for 10-year survival is approximately 50%, although the tumor growth rate in individual patients has shown great variability. Patients with the MEN IIb disease require total thyroidectomy as soon as the syndrome is recognized, preferably by the age of 2 years. In familial cases, the characteristic findings are sufficient even without calcitonin testing to justify operation. If the diagnosis is not made until adolescence or later, both central compartment and lateral node involvement require neck dissection for definitive treatment.


46. Which of the following statements regarding thyroid physiology are true?

a. Normally about 20% of T3 is secreted directly from the thyroid gland
b. The thyroid gland is the only endogenous source of T4
c. Excess thyroid hormone results in an increase in the number of ATP-dependent sodium pumps on the cell membrane
d. The majority of thyroid hormone in circulating plasma is bound to albumin, thus limiting the availability of the metabolically active form
Answer: a, b, c

The thyroid is the only endogenous source of T4, whereas most T3 is produced by the peripheral conversion of T4. This takes place in the liver, muscle, kidney and anterior pituitary. Under normal circumstances, only about 20% of T3 is secreted directly from the thyroid gland. In some thyroid diseases (e.g., Graves’ disease and toxic nodular goiter), the proportion of T3 secreted directly by the thyroid may be markedly increased.
Once thyroid hormones are released into the circulation, they are bound to thyroid-binding globulin (85%), albumin (10%) and transthyretin (prealbumin). These binding proteins allow the thyroid hormones to remain soluble in plasma, contributing to systemic distribution to various target-cell populations. A limited amount of thyroid hormones circulates freely in the plasma in metabolically active form (free T4, free T3).
At the cell membrane, an excess of thyroid hormone results in an increased number of ATP-dependent sodium pumps, thus increasing resting energy expenditure and oxygen consumption. Thyroid hormone also facilitates the transport of glucose and amino acids across the cell membrane. In addition, T3-induced mitochondrial oxidation of substrate results in increased ATP production.


47. The definitive treatment of choice for toxic multinodular goiter is?

a. Total thyroidectomy
b. Bilateral subtotal thyroidectomy
c. Unilateral total lobectomy on the side of dominant disease
d. 131I treatment
Answer: b

Hyperthyroidism in the patient with a multinodular goiter usually develops in women after age 50 but is seen occasionally in younger patients. Most patients have had a nontoxic nodular goiter for many years. Preferred treatment for most patients with toxic multinodular goiters is thyroidectomy after adequate preparation renders the patient euthyroid. 131I may be an alternative in selected poor-risk patients with goiters that are not causing airway compression. Although 131I can be used to treat the hyperthyroidism, larger and often repeated doses of 131I may be required. 131I does not significantly reduce the goiter size and may, because of radiation-induced thyroiditis, cause acute enlargement. This may be hazardous in the patient with some degree of preexisting airway compression. Any airway symptoms, particularly in patients with substernal goiters, should be considered strong contraindications to the use of 131I.
Standard surgical treatment of toxic nodular goiter has consisted of bilateral subtotal thyroidectomy. Remnant size is not as important as the excision of all autonomous nodules. Because thyroid replacement or suppression is used routinely to prevent recurrence of goiter when a subtotal resection is done, the risk of hypothyroidism is not a consideration in determining remnant size. Alternative procedures are total lobectomy with isthmus resection and contralateral subtotal lobectomy, or total thyroidectomy. The latter is not demonstrably superior and may have more technical complications.


48. A 50-year-old male has undergone an ipsilateral thyroid lobectomy and isthmus resection for what appeared on frozen section to be a benign nodular lesion 2.0 cm in diameter. Seventy-two hours later the final pathology returns and the diagnosis is a high-grade angioinvasive follicular carcinoma. What do you recommend?

a. 99mTc bone scan to rule out occult bone metastases
b. Ipsilateral radical neck dissection
c. Total thyroidectomy
d. Observation with sequential 131I scans every 3 months
Answer: c

Follicular carcinoma accounts for about 10% of all new carcinomas of the thyroid. Most patients are those with minimal invasion of the capsule or vessels within the neoplasms. Such tumors are seldom diagnosed definitively by either needle aspiration cytology or by frozen-section diagnosis at the time of lobectomy. Most frequently, the diagnosis is made after the study of permanent sections. Microinvasive encapsulated follicular carcinomas are rarely associated with metastatic lymph nodes and distant metastases involving bone are also rare at the time of diagnosis.
Angioinvasive follicular carcinomas are usually large and frequently show venous invasion of perithyroidal and lateral neck veins at the time of diagnosis. They may have already metastasized to different sites, most frequently the bone. These tumors are often diagnosed at the time of operation because of these characteristics. Most patients younger than 40 do well, but patients older than 50 years have a guarded prognosis. Follicular carcinomas of the thyroid are treated by total thyroidectomy. Lymphatic dissections are not usually required because only about 5% of all patients have lymphatic involvement. The most effective therapy for bone or pulmonary metastases is radioactive iodine. The controversy in the surgical management of follicular carcinoma is whether patients with low-grade encapsulated neoplasms diagnosed after total lobectomy by permanent section evaluation should be treated with total thyroidectomy. In patients with smaller follicular neoplasms found malignant as determined by microinvasion of the capsule, completion thyroidectomy is usually not done; instead patients are observed closely after performing a 99mTc bone scan to rule out occult bone metastases. Patients with large or high-grade angioinvasive lesions require total thyroidectomy.

49. In 1990 a National Institutes of Health Consensus Development Conference reviewed the available evidence regarding the management of asymptomatic primary hyperparathyroidism. The panel concluded that surgical intervention was indicated for which of the following indications?

a. Age less than 50 years of age
b. Reduced creatinine clearance
c. Presence of kidney stone(s) (as detected by abdominal radiograph)
d. Substantial reduced bone mass as determined by direct measurement
e. Markedly elevated 24-hour urinary calcium excretion
Answer: a, b, c, d, e

All of the indications listed above are considered indications for operative treatment of asymptomatic patients with primary hyperparathyroidism. Their conclusions regarding operative indications are summarized in Table 57-8. The NIH Consensus Development Conference mandated close (every 6 months) follow-up for patients with known primary hyperparathyroidism not treated by operation. In addition, surgery was agreed upon for those patients in whom medical surveillance was neither desirable nor suitable, such as when the patient requests surgery, consistent follow-up is unlikely, co-existent illness complicates management, or if the patient is young (< 50 year of age). In one recent study of a group of 142 asymptomatic patients followed without operation, after 10 years, more than 20% of the patients had required surgery for an increase in serum calcium to greater than 11 mg/dL or for specific complications attributable to the disease. Another 20% were lost to or declined follow-up. The remainder either died of unrelated causes or had persistent asymptomatic disease. This remains an area of considerable controversy. The best available recommendations are derived from this October 1990 NIH Consensus Development Conference. 50. Which of the following statements regarding medullary carcinoma of the thyroid are true? a. Bilateral medullary carcinoma of the thyroid occurs in every patient with multiple endocrine neoplasia 1, 2a or 2b b. Medullary carcinoma of the thyroid accounts for approximately one-third of all thyroid malignancies c. 20% of medullary carcinoma of the thyroid cases are familial d. Provocative testing with pentagastrin and calcium infusion for family members at risk for medullary carcinoma of the thyroid is diminished in importance by the recent development of specific genetic testing Answer: a, c, d Medullary thyroid carcinoma accounts for about 10% of all thyroid malignancies, and 20% of cases occur in the familial setting of MEN 2a, MEN 2B, or familial non-MEN medullary thyroid carcinoma. It is usually the first tumor that develops in these patients and typically occurs in the second or third decade of life. Tumors are virtually always bilateral and develop in multiple areas of the middle and upper portions of the thyroid lobe. Detection has historically been based on elaboration of calcitonin, particularly in response to provocative testing by the potent secretagogues calcium and pentagastrin. Patients with medullary carcinoma of the thyroid have striking increases in plasma calcitonin levels after provocative testing, which allows them to be readily identified. By employing provocative testing in kindred members at risk for disease, medullary carcinoma of the thyroid was diagnosed at a preclinical stage, and a greater percentage of these patients were cured by surgical therapy. With genetic testing now available, early treatment of medullary thyroid cancer should be possible for all affected people, to hopefully enhance the number of people cured of this cancer. The MEN 1, 2a and 2b syndromes are inherited in an autosomal dominant fashion with complete penetrance but variable phenotype. Bilateral medullary carcinoma of the thyroid occurs in every affected patient. 51. While awaiting surgery on a HMO waiting list, a 50-year-old female with primary hyperparathyroidism is admitted to the hospital with oliguria, confusion, nausea and vomiting, muscle weakness and a serum calcium of 13.5 mg/dL. Of the treatment options below, which one is the most appropriate? a. Administer 1 gm of hydrocortisone STAT b. Begin an IV EDTA (chelating agent) infusion STAT c. Administer IV Mithramycin and calcitonin concurrently and proceed to ICU for cardiac monitoring d. Begin a normal saline infusion at 2X maintenance volume followed by 1mg/kg furosemide IV e. None of the above Answer: d Patients with marked hypercalcemia or severe symptoms should be admitted to the hospital for careful observation and monitoring. The mainstay of therapy is intravenous hydration, preferably with normal saline in sufficient quantities to maintain the urine output above 100 mL/h. These patients are often dehydrated before therapy, and fluid can be administered intravenously at a rate of 200 mL/h. Caution must be exercised in older patients who might have marginal cardiac reserve. The diuretic furosemide also increases excretion of sodium and calcium but should not be employed until the patient is well hydrated. Saline diuresis is usually effective when the hypercalcemia results from hyperparathyroidism or from a benign cause. In contrast, the hypercalcemia of malignancy may produce severe symptoms associated with extremely high serum calcium levels that are difficult to control. In this setting, a variety of other measures may be considered (see Table 57-3). Some of the agents used to treat hypercalcemia have significant toxicity and require close monitoring. Calcitonin is a fairly weak hypocalcemic agent, but it acts rapidly and is relatively less toxic. Glucocorticoids may be particularly efficacious in patients with sarcoidosis and other granulomatous diseases. Mithramycin has proved useful in patients with hypercalcemia of malignancy, but it has a substantial cumulative toxicity (thrombocytopenia, hepatotoxicity, and nephrotoxicity). Intravenous phosphates and chelating agents have largely been abandoned because of their severe toxicity. 52. Which of the following conditions are associated with hypercalcemia? a. Hyperthyroidism b. Thiazide diuretics c. Terminal ileal resection d. Breast cancer e. All of the above Answer: a, b, d Multiple causes of hypercalcemia include the following: Malignancy Vitamin A or D intoxication Thiazide diuretics Hyperthyroidism Milk-alkali syndrome Sarcoidosis and other granulomatous diseases Familial hypocalciuric hypercalcemia Immobilization Paget’s disease Lithium therapy Addisonian crisis Idiopathic hypercalcemia of infancy Hyperthyroidism is associated with increased bone reabsorption. Often, the plasma PTH is low, and a history of other thyrotoxic symptoms can be elicited. Thiazides may increase serum calcium to a mild degree, primarily through hemoconcentration. Serum phosphate may also be depressed. It often takes several weeks for the hypercalcemia to resolve after the medication is discontinued. Terminal ileal resection is not associated with hypercalcemia, indeed hypocalcemia may develop with short gut syndrome. While ileal resection is a predisposing condition for the development of calcium oxalate kidney stones, this is the result of alterations in oxalate metabolism. Patients with solid tumors, such as lung carcinoma, breast carcinoma, squamous cell carcinoma of the head, neck, esophagus or female genital tract or renal cell cancer account for three quarters of all cases of humoral hypercalcemia of malignancy. 53. A 40-year-old male undergoes an apparently uneventful total thyroidectomy for follicular carcinoma of the thyroid. 48 hours later he develops circumoral numbness, followed by laryngospasm, and then has a generalized seizure. Of the following, which is the first priority? a. Proceed to OR for exploration of the operative site b. Administer 25 ml of 10% calcium gluconate intravenously c. Obtain a serum magnesium measurement and administer intravenous magnesium chloride STAT d. Obtain a CT scan of the head to evaluate the possibility of brain metastases Answer: b Postoperative hypoparathyroidism commonly develops after total thyroidectomy for malignancy. Most patients undergoing operation on the thyroid experience some alteration in serum calcium. The hypocalcemia is usually transient and is not treated unless significant symptoms develop. The plasma calcium usually reaches its nadir at 48 to 72 hours after surgery and then slowly returns to normal over 2 to 3 days. For these reasons, careful postoperative monitoring of serum calcium levels is essential. For acute symptomatic hypocalcemia, calcium should be administered intravenously. Usually 20 to 30 mL of 10% calcium gluconate is infused over a 15 to 20 minute period and then 50 to 100 mL are administered over the next 12 hours in adults. Calcium gluconate is less irritating to the veins than calcium chloride, and the calcium release is slower without the risk of overcorrection. Serum magnesium should always be measured and hypomagnesemia should be corrected if present. This is not the first priority for this patient. Symptoms should never be allowed to progress to the point demonstrated in this patient. Postoperative routines include careful monitoring of the serum calcium until stable. There is no evidence that this patient has a postoperative surgical problem such as a cervical hematoma. Laryngospasm and seizures are classical signs of hypocalcemia. Lastly, the likelihood of metastatic brain disease is very low in this circumstance. 54. On routine life insurance screening, an asymptomatic 45-year-old male is found to have a serum calcium level of 12.5 mg/dL. Serum is obtained for immunoreactive parathyroid hormone level and this is 400 mEq/mL (normal range < 64mEq/mL). These findings are most consistent with a diagnosis of which of the following? a. Primary hyperparathyroidism b. Secondary hyperparathyroidism c. Tertiary hyperparathyroidism d. Ectopic hyperparathyroidism Answer: a Primary hyperparathyroidism develops spontaneously without apparent cause, but possibly in response to exogenous stimuli. When the normal control of serum calcium is disturbed and there is increased autonomous production of PTH, the state is referred to as primary hyperparathyroidism. In contrast, secondary hyperparathyroidism occurs when there is a defect in mineral homeostasis that leads to a compensatory increase in parathyroid function. This occurs most commonly in response to renal disease, but may also develop as a consequence of the hypocalcemia associated with some diseases of the gastrointestinal tract, bone or other endocrine organs. Occasionally, with prolonged secondary stimulation, the hyperfunctioning glands are no longer physiologically responsive to an increased ionized calcium. This rare, relatively autonomous state is referred to as tertiary hyperparathyroidism and develops most commonly after renal transplantation when the defect in calcium homeostatis is corrected. The numerical values for calcium and PTH here are consistent with primary hyperparathyroidism. Relation between serum immunoreactive parathyroid hormone (iPTH) and serum calcium in patients with hypoparathyroidism, pseudohypoparathyroidism, ectopic hyperparathyroidism, and primary, secondary, and tertiary hyperparathyroidism. GP1M, guinea pig antiserum 1M. (After Clark OH, Way LW. Thyroid and parathyroid. In: Current surgical diagnosis and treatment, ed 8. Norwalk, CT, Appleton & Lange, 1989:249) 55. A 50-year-old female is referred for treatment of a serum calcium of 11.5 mg/dL one year after resection of a right superior parathyroid adenoma. The right and left inferior glands were normal at the initial neck exploration. The left superior gland was not visualized. Which of the related statements below are true? a. Recurrent postoperative hypercalcemia occurs in approximately 20% of patients with this clinical scenario b. Localization studies via selective angiography are successful in 50% to 80% of these patients c. The risk of permanent hypoparathyroidism is approximately 10% to 20% following reexploration in this setting d. Surgical reexploration by an experienced endocrine surgeon has a success rate of less than 50% in this circumstance Answer: b, c Persistent hyperparathyroidism occurs in less than 5% of patients after neck exploration for primary hyperparathyroidism by an experienced surgeon. Most commonly, it is the result of a single diseased gland still remaining in the neck or in the mediastinum. Recurrent disease develops after an interval of normocalcemia and may be the result of regrowth of diseased tissue, implantation from a tumor broken at the initial procedure, or even recurrent parathyroid carcinoma. Documentation of a correct initial diagnosis and review of the original operative and pathology reports are essential. It is generally agreed that localization studies do have a place in the management of recurrent disease. Noninvasive methods are employed first, and if these are unsuccessful in identifying the diseased gland, selective angiography and venous sampling for PTH are employed. Selective angiography appears to be the most accurate technique, successfully localizing 50% to 80% of parathyroid glands that cannot be detected by any other modality. Surgical reexploration can be difficult. Generally the neck is explored first after which the mediastinum is examined if this is unproductive. Surgical reexploration is successful in experienced hands in 60% to 80% of cases. There is, however, an increased incidence of complications. Unilateral recurrent nerve injury occurs in 5% to 10% of patients and permanent hypoparathyroidism occurs in 10% to 20% of patients postoperatively. 56. Which of the following statements regarding calcium and phosphate metabolism are true? a. Parathyroid hormone excess produces a net increase in daily urinary calcium excretion b. Calcitonin is essential for the normal control of calcium metabolism in adult humans c. Parathyroid hormone is the single most important regulator of calcium and phosphate metabolism in humans d. 1,25 dihydroxyvitamin D3 (calcitriol) is the active form of vitamin D in humans Answer: a, c, d The primary hormonal regulators of calcium and phosphate metabolism are parathyroid hormone (PTH), vitamin D and calcitonin. Parathyroid hormone appears to be the single most important hormonal regulator of calcium and phosphate metabolism in humans. It has direct effects on the skeleton and kidney and indirect effects on the intestine, mediated through vitamin D. Sustained elevations of PTH stimulate osteoclasts and inhibit osteoblasts leading to absorption of calcium from bone. In the kidney, PTH produces an increase in reabsorption at any given concentration of extracellular fluid calcium, although excess secretion, because of the hypercalcemia, results in a net increase in daily urinary calcium excretion. Vitamin D3, or cholecalciferol, is produced normally by the action of sunlight on 7-dehydrocholesterol in the skin. It is then hydroxylated in the liver (25 position) and kidney (1 position) to form the active 1,25 dihydroxyvitamin D3 (calcitriol). This is the active form of Vitamin D in humans. Calcitonin is a 32-amino acid protein produced by the parafollicular C cells of the thyroid. Total thyroidectomy, with removal of all the C cells, is well tolerated, and it has been concluded that calcitonin is not essential for the normal control of calcium metabolism in adult humans. Calcitonin does inhibit bone resorption and can produce hypocalcemia in experimental animals. It also increases urinary calcium and phosphate excretion. 57. Multiple Endocrine Neoplasia (MEN) 2b is characterized by which of the following findings? a. Medullary carcinoma of the thyroid, pheochromocytoma, mucosal neuromas, and a distinctive marfanoid habitus b. Parathyroid hyperplasia, pancreatic islet cell tumors, and pituitary adenomas c. Medullary carcinoma of the thyroid, pheochromocytoma, and parathyroid hyperplasia d. Parathyroid carcinoma, pheochromocytoma and chronic pancreatitis Answer: a The multiple endocrine neoplasias are familial disorders typically characterized by predisposition to the development of tumors of multiple endocrine organs. These disorders are all inherited in an autosomal dominant fashion, and the tumors tend to be multicentric. The tumors may be benign or malignant and may occur metachronously or synchronously. MEN 1 is characterized by the concurrence of parathyroid hyperplasia, pancreatic islet cell tumors, and pituitary adenomas. MEN 2a consists of medullary carcinoma of the thyroid, pheochromocytoma, and parathyroid hyperplasia. MEN 2b includes medullary carcinoma of the thyroid, pheochromocytoma, mucosal neuromas, and a distinctive marfanoid habitus. Together, these syndromes encompass much of the spectrum of endocrine neoplasia. 58. Which of the following signs/symptoms are pathognomonic of hyperparathyroidism? a. Pathologic fractures of the metacarpals b. Calcium oxalate nephrolithiasis c. Hypercalcemia causing mental status changes d. Atrophy of Type II muscle fibers e. Osteitis fibrosa cystica Answer: e Manifestations of hyperparathyroidism are protean, but generally nonspecific and may be difficult to elicit in the history. The earliest complaints are often vague and include muscle weakness, anorexia, nausea, constipation, polyuria, and polydipsia. These nonspecific symptoms may or may not cause the patient to seek medical attention. Symptomatic patients generally have evidence of chronic disease involving the kidney or the skeleton. Renal complications develop because the hypercalcemia leads to increased urinary calcium excretion and PTH increases the excretion of phosphate. These events predispose to stone formation. Nephrolithiasis develops in about 30% of patients. Nephrocalcinosis occurs in 5% to 10% of patients. These are both nonspecific for hyperparathyroidism. Parathyroid bone disease in its most classic and severe form, osteitis fibrosa cystica, is seldom seen. However, 5% to 15% of patients present with significant symptoms of skeletal disease. Only the skeletal disease is pathognomonic. Bone changes include subperiosteal resorption of bone on the radial aspect of the middle phalanx of the second or third finger. Tufting of the distal phalanges, bone cysts of the skull and long bones, “brown” tumors (i.e., localized proliferations of osteoclasts), and diffuse demineralization or granularity are seen as well. Atrophy of Type II muscle fibers, consistent with a neuropathic and not a myopathic cause, has been demonstrated in patients with hyperthyroidism, but this too is nonspecific. 59. The causes of Cushing syndrome may include which of the following? a. Posterior pituitary adenoma b. Adrenal hyperplasia c. Small cell lung carcinoma d. Pheochromocytoma e. Adrenal carcinoma Answer: b, c, e The varied causes of cortisol excess produce clinical features that are collectively called Cushing syndrome. These include exogenous steroid administration, Cushing disease (excessive ACTH production by the anterior pituitary gland, usually from an adenoma), ectopic ACTH production (small cell lung carcinoma), adrenal adenoma or carcinoma, micronodular pigmented hyperplasia, macronodular hyperplasia, and steroid-dependent adrenal hyperplasia. Pheochromocytoma is characterized by catecholamine rather than cortisol excess as it arises from the adrenal medulla rather than the adrenal cortex. 60. Which of the following are normal systemic effects of glucocorticoids? a. Enhanced proteolysis b. Increased gluconeogenesis c. Diminished lipolysis d. Decreased rate of intestinal epithelial replication Answer: a, b, d The many systemic effects of glucocorticoids are related to regulation of intermediary metabolism. In this regard, perhaps the most important action is the effect of steroids on protein breakdown. A direct proteolytic effect of steroids is suggested by several lines of evidence. Glucocorticoids enhance gluconeogenesis by both a direct effect on gluconeogenic hepatic enzymes and also by provision of substrate for gluconeogenesis by proteolysis. Glucocorticoid influence leads to the accentuation of lipolysis. The truncal obesity seen in steroid excess is related to the predominance of the lipogenic effect of insulin on truncal adipocytes over the lipolytic effect of glucocorticoids. The opposite relation may hold for the receptors in fat of the extremities and would explain the comparatively scant fat in these areas with steroid excess. The most notable effect of glucocorticoids in the intestinal tract is a decrease in the rate of mucosal cell replication. In addition, decreased mucosal and pancreatic prostaglandin synthesis occur. This may have important implications for the cytoprotective mechanisms in the stomach. 61. A 10-year-old child presents with hypertension, tachycardia, nervousness and sweating. The best initial diagnostic evaluation is which of the following? a. Radioimmunoassays for norepinephrine and epinephrine in serum b. Magnetic resonance imaging of the adrenal gland c. MIBG (131I-Methaiodobenzylguanidine) scintigraphy d. Measurement of catecholamines and their degradation products in a 24-hour urine specimen Answer: d The first diagnostic step in determining the functional state of an adrenal gland or lesion is to screen the urine or plasma for secretory products. Once hypersecretion is demonstrated, the specific type of pathology producing a syndrome must be determined with the aid of functional tests and relevant scanning and imaging. The most efficient and sensitive means of screening in a patient suspected of having a pheochromocytoma, (as indicated here) is measurement of the catecholamines or metabolic products thereof in the urine. Although 24-hour samples can smooth out the possible episodic variations in catecholamine secretion, shorter sampling periods can be useful, especially if corrected for creatinine excretion. Timing of the collection is critical in patients who have only episodic hypertension. Urine collection should be started immediately after a suspected attack of hypertension. Fluctuations in plasma catecholamine concentrations are much greater than those in urinary excretion, even in normal subjects. Plasma determinations are quite sensitive and specific with radioimmunoassays and HPLC determination of plasma catecholamines, but the specificity can be low because of the overlap of normal spikes in catecholamine concentrations with concentrations produced by minimally secreting pheochromocytomas. MRI and MIBG imaging studies are both potentially useful after catecholamine excess is confirmed. MRI demonstrates anatomy quite well. MIBG scintigraphy is particularly useful in looking for nonadrenal and bilateral pheochromocytomas. 62. A 25-year-old male has been taking 40 mg of prednisone PO qod for ulcerative colitis for 5 years. He undergoes an uneventful colectomy with endorectal pull-through and an ileoanal anastomosis. Which of the following statements regarding steriod management are correct? a. On the day of surgery he should receive 100 mg hydrocortisone IV q6h b. The postoperative steroid dose should be halved q 12 hours to reduce the risk of infectious complications and improve would healing c. Prophylactic treatment with a somatostatin analogue will reduce the risk of postoperative pancreatitis d. Exogenous steroid replacement can be stopped after 3 months Answer: a The postoperative course of management for a patient with pituitary adrenal suppression from exogenous steroids involves tapering the exogenous steroid doses to maintenance levels after high dose replacement at the time of operation. One simple regimen involves administering 100 mg of hydrocortisone intravenously every 6 hours during the first 48 hours. Some prefer alternating doses of IM cortisone acetate in the event that intravenous access is lost. Provided that no intervening complications arise, the doses can be halved every 48 to 72 hours. In patients who have been exposed preoperatively to glucocorticoid excess, the maintenance dose may be as high as 100 mg/d for several months. Both high doses and normal maintenance of 35 to 55 mg/d can be given in the form of oral cortisone acetate as long as reliable alimentation and absorption have been achieved. The pituitary-adrenal axis remains suppressed for 6 to 12 months after operation. Complications in the postoperative period include wound infection, pancreatitis, and thromboembolism. There is no data to suggest that the risk of postoperative pancreatitis can be diminished with somatostatin analogue therapy. 63. Imaging of the adrenal gland is best achieved with which of the following techniques? a. Ultrasound b. Computed tomography (CT) c. Arteriography d. Scintigraphy with 131I-6 b-iodomethyl-19-norcholesterol (NP-59) e. Scintigraphy with 131I-methaiodobenzylguanidine (MIBG) Answer: b Although ultrasonography is the least expensive of the imaging procedures, its value is limited by the relative inaccessibility of the adrenal gland and by the small size of some adrenal lesions. CT is the technique most commonly used to examine patients in whom adrenal abnormalities are suspected. CT reliably detects adrenal tumors greater than 1 cm in diameter. The sensitivity of CT for tumors that are 1 cm in diameter is about 80% and reaches 100% for tumors that are 3 to 4 cm. Although CT is noninvasive and reasonably sensitive, it is nonspecific. It does not distinguish functioning from nonfunctioning tumors, nor benign from malignant tumors with any degree of reliability. MRI has developed a certain usefulness even after retrenchment from early optimistic predictions. MRI is more expensive and requires greater patient cooperation than CT, but it has greater versatility because of the use of TI-and T2-weighted images. In some cases, the T2-weighted images are capable of providing a differential diagnosis and may distinguish such entities as metastatic or primary carcinoma and pheochromocytoma from adenomas, lipomas, myelolipomas and cysts. In a sense, MRI is complementary to CT in that the latter can better detect the lesion while the former can distinguish one type of lesion from the other. In addition, MRI is probably better than CT for distinguishing anatomic relationships and the extent of involvement of the surrounding tissues. Two radiopharmaceuticals have proved useful in imaging the adrenal gland. Adrenocortical lesions can be imaged by NP-59 which is taken up as cholesterol in the adrenocortical steroidogenic pathway. The other agent is MIBG, a norepinephrine analogue. It indicates norepinephrine accumulation in storage vesicles and can detect sympathoadrenal tumors at any site in the body. NP-59 can localize the adrenal cortex and any functioning tumors. It can distinguish adrenocortical hyperplasia from functioning adenomas or carcinomas. MIBG is a useful agent in localizing pheochromocytomas throughout the body, especially when the tumors are multiple, extraadrenal, recurrent, or metastatic. Arteriography, venography and selective venous sampling have become less popular as experience with other imaging techniques has become greater. Disadvantages inherent in invasive procedures using intravascular contrast agents are obvious. Arteriography is specifically dangerous in the study of patients with pheochromocytomas. 64. A 45-year-old female is found to have a 2 cm solid nodule in her right adrenal gland at the time of an abdominal CT scan following an auto accident. With regard to the adrenal lesion, she is asymptomatic and it is found to be nonfunctional on evaluation. You would recommend which of the following? a. Extraperitoneal right adrenalectomy through either a flank of posterior approach b. Suppression with 5 mg prednisone PO qod c. Followup CT scan in 1 to 3 months d. Excisional biopsy via laparaoscopic approach Answer: c Indication for operation in the patient with a unilateral functioning adrenal tumor is clear. In the patient with a nonfunctioning adrenal tumor, the need for surgery is related to the size of the tumor and its rate of growth. There is consensus that a tumor larger than 6 cm should be removed. Some recommend that the acceptable size limit be 3 cm, especially when MRI suggests carcinoma or when the functional studies suggest activity. When nonoperative therapy is elected, the patient should receive an adrenal scan 1, 3, and 6 months after the initial scan and yearly thereafter to assess the growth of the lesion. If the tumor has grown, surgical removal is indicated. 65. A 20-year-old male with a 10 cm left adrenal mass is found to have 10 mg of norepinephrine in a 24-hour urine collection and a plasma 18 hydroxycorticosterone level of 50 mg/dL. Initial reoperative preparation should include which of the following? a. Treatment with spironolactone b. Intravenous potassium-loading to prevent intraoperative hypokalemia c. Treatment with phenoxybenzamine d. Treatment with labetalol Answer: c This patient has a pheochromocytoma. The most efficient and sensitive means of screening for pheochromocytoma is measurement of the catecholamines, or metabolic products thereof, in the urine. The catecholamines, norepinephrine and epinephrine, are excreted in amounts under 100 µg/d in the normal person. Because of some overlap in values, specificity can be improved by using a normal range of up to 250 mg/d. The measurement of plasma 18-hydroxycorticosterone level is helpful in evaluating patients with hyperaldosteronism, as it is an intermediate product in its synthesis. The 18-hydroxycorticosterone levels are above 100 mg/dL in virtually all patients with aldosterone producing adenomas. The plasma value for this patient is normal. Perioperative treatment with either spironolactone and potassium replacement is appropriate for patients with hyperaldosteronism, but not pheochromocytoma. Nonoperative treatment of pheochromocytoma is generally unsatisfactory and entails pharmacologic blockade of the effects of catecholamines. Phenoxybenzamine and prazosin are two preferred agents that block the a-adrenergic effects of the catecholamines preoperatively with pheochromocytoma. The use of b-adrenergic blockers, such as labetalol, may be required in those patients with obvious b-adrenergic effects, such as resting pulse rates above 100 beats/min. Because of the potential for wide swings in blood pressure and other effects of chronic catecholamine secretion, careful preoperative preparation is required in patients with pheochromocytoma. It is customary to institute a-adrenergic blockade 2 to 3 weeks before anticipated surgery. This has beneficial effects of controlling blood pressure and allowing restoration of a decreased blood volume. It is the consensus that preoperative preparation in the manner makes the intraoperative management of the patient much more safe. In patients who require b-adrenergic blockade, it is essential to first establish good a-adrenergic blockade. These patients are prone to cardiac failure induced by b-adrenergic blockade because of the cardiomyopathy that may preexist. b-Adrenergic blockade in the cardiomyopathic patient with failure to first reduce the afterload by a-adrenergic blockade, can precipitate cardiac failure. 66. Which of the following diagnostic tests will differentiate between the pituitary and adrenal gland as the cause of hypercortisolism? a. High dose dexamethasone suppression test b. CRH stimulation test c. Low dose dexamethasone test d. AM and PM plasma cortisol determination Answer: a, b Diurnal variation of plasma cortisol levels is lost in hypercortisolism, caused by both adrenal tumors and pituitary lesions. Dexamethasone, by negative feedback, suppresses the hypothalamic pituitary secretion of ACTH and effects the consequent lowering of both plasma cortisol and urinary 17-hydroxycorticosteroid excretion. Administration of a single dose of 2 mg of dexamethasone suppresses plasma cortisol and urinary 17-hydroxycorticosteroids by at least half when compared to control values from untreated normal patients (low dose suppression test). In Cushing disease, with a setpoint of ACTH secretion higher than normal, low-dose dexamethasone is insufficient to suppress ACTH. High-dose dexamethasone suppression is achieved by 2 mg of dexamethasone administered every 6 hours for 24 hours. A normal response is to lower the 17-hydroxycorticosteroid excretion by more than one half. In the case of Cushing disease, the hypothalamic steroid receptors that allow negative feedback are intact but at a higher setpoint. In this case, 17-hydroxycorticosteroid secretion does decrease significantly after high-dose dexamethasone administration. Adrenal tumors, other causes of ectopic production of ACTH, and most cases of nodular hyperplasia do not respond to dexamethasone suppression with a decrease in steroid secretion. With an adrenal tumor, pituitary ACTH is already suppressed; therefore, dexamethasone cannot suppress it further. With ectopic ACTH secretion, the tissue producing ACTH has no receptors for steroids and thus negative feedback cannot be achieved. Therefore, high dose dexamethasone suppression will differentiate hypercortisolism of pituitary and adrenal origins. Potentially the most helpful new test for this purpose uses the administration of recombinant CRH to release ACTH and consequently to stimulate cortisol secretion. CRH (1 mg/kg) is administered intravenously, and serial blood samples obtained for 3 hours after administration. The normal pituitary adrenal axis responds by a moderate increase in ACTH and cortisol. With Cushing disease, the ACTH and cortisol rise are accentuated. With adrenal autonomous production of cortisol and with ectopic ACTH production, there is virtually no response to CRH. 67. Which of the following statements regarding androgens and estrogens in the fetus are true? a. The development of normal female external genitalia requires estrogen production by the ovary b. A female with congenital adrenal hyperplasia is likely to be masculinized in appearance c. The development of normal male external genitalia requires adrenal androgen production d. A male with congenital adrenal hyperplasia is likely to be feminized in appearance Answer: b, c Adrenal androgens in the fetus stimulate wolffian duct development and elongate the genital tubercle. They promote midline migration of the labial folds and fusion of these folds to form the scrotum. To complete the male transformation, the urethral opening migrates to the tip of the phallus. All of these events are androgen-dependent. Since the normal female fetus does not secrete androgens, the genital tubercle, labial folds, and urethral opening all remain in the female position in this circumstance. Thus the female phenotype is associated with the absence of fetal sex hormone production. Excess androgen in the female fetus causes neonatal virilization, as is seen with congenital adrenal hyperplasia. The male infant with congenital adrenal hyperplasia is likely to have a normal appearance of the external genitalia as a neonate. Precocious puberty will develop over a period of years in this latter circumstance. 68. Which of the following statements regarding the physiology of the adrenal gland are true? a. Release of CRH is regulated principally by negative feedback by ACTH b. Plasma 17-ketosteroid levels reflect the degree of adrenal cortisol production c. Renin undergoes enzymatic cleavage in the lung to angiotensin I d. The plasma half-life of ACTH is relatively long (> 24 hrs)
e. None of the above
Answer: e

The proximate stimulator of cortisol production is the peptide hormone, andrenocorticotropic hormone (ACTH). It originates from the anterior pituitary gland and is regulated by corticotropin-releasing hormone (CRH). The regulation of CRH is controlled by various neural influences. These include intrinsic central nervous system influences and a negative feedback inhibition by cortisol. Although there is some evidence of a short-loop feedback of ACTH on CRH, both slow and fast feedback by cortisol on the pituitary release mechanism are the primary sources of clinically relevant CRH regulation.
The steroidogenic pathway involves the conversion of cholesterol to pregnenolone, progesterone, 17-hydroxyprogesterone, and then either to the adrenal androgens or cortisol via several intermediates. The 17-ketosteroids reflect adrenal androgen synthesis while the 17-hydroxysteroids reflect cortisol synthesis.
Renin is produced predominantly in the juxtaglomerular apparatus of the kidney where it acts locally and is released into the systemic circulation. Renin cleaves angiotensin I, a decapeptide derived from the liver which serves as renin substrate. Angiotensin I undergoes enzymatic cleavage in the lung to angiotensin II, which is the biologically active form of the peptide.
The plasma half-life of ACTH is short (measured in minutes) with a rapid onset of action. This is in contrast to a longer plasma half-life and a slower onset of action for cortisol itself.


69. The approximate 5-year survival rate for adrenocortical carcinoma is which of the following?

a. 0%
b. 20%–25%
c. 50%–60%
d. Nearly 100%
Answer: b

The prognosis for adrenocortical carcinoma is not good. The overall 5-year survival rate is 20% to 25% for these malignancies. When there is localized disease at the time of surgery, the 5-year survival may be higher, in the 40% to 50% range. The true prognosis in childhood is not clear, but the data suggest a 2-year survival rate of about 20%.


70. Which of the following statements is true with respect to pheochromocytoma?

a. Pheochromocytoma associated with MEN IIa is usually unilateral and rarely malignant; therefore, unilateral exploration through a posterior flank incision is usually sufficient
b. Clonidine fails to suppress basal plasma-catecholamine levels in patients with pheochromocytoma
c. NP-59 (131I-6 b-iodomethyl-19-norcholesterol) is taken up as cholesterol by the adrenal medulla
d. The ratio of plasma 3,4-dihydroxyphenoglycol (DHPG) to norepinephrine is generally elevated in patients with pheochromocytoma compared to patients with essential hypertension
Answer: b, d

Pheochromocytoma associated with the MEN IIa syndrome is more often bilateral and more often malignant, therefore, abdominal exploration through an anterior approach is indicated. The ability to measure catecholamines in the plasma has made possible the clonidine suppression test. In patients without pheochromocytoma, clonidine suppresses high basal plasma concentrations into the normal range, whereas concentrations in patients with pheochromocytoma are not suppressed. Another use of plasma catecholamine measurements is in examining the ratio of 3,4-dihydroxyphenoglycol (DHPG) to norepinephrine in plasma. DHPG is released from the chromaffin cells and adrenergic neurons to a much greater extent than norepinephrine in pheochromocytoma patients compared with patients who have essential hypertension, i.e. the ratio of DHPG to norepinephrine is higher in patients with pheochromocytomas.
NP-59 (131I-6 b-iodomethyl-19-norcholesterol) is taken up as cholesterol by the adrenal cortex and is incorporated in the adrenocortical steroidogenic pathway. This is a useful agent for imaging adrenocorticol lesions. 131I-methaiodobenzylguanidine (MIBG) is a norepinephrine analogue that is useful in localizing pheochromocytomas throughout the body, especially when the tumors are multiple, extraadrenal, recurrent, or metastatic.


71. Which of the following statements regarding aldosterone are true?

a. Its secretion is directly related to the serum potassium concentration
b. Angiotensin II is a more potent regulatory factor than ACTH
c. Primary hyperaldosteronism is characterized by hyperkalemia
d. Secondary hyperaldosteronism occurs with renal artery stenosis
Answer: a, b, d

Aldosterone secretion is controlled by changes in the afferent arteriolar pressure in the renal cortex as well as by changes in sodium content in the renal tubule. These changes are sensed by the juxtaglomerular apparatus and by the macula densa and act through the renin angiotensin system. At least two other factors influence aldosterone secretion. Aldosterone secretion is directly related to the serum potassium concentration. An increase in serum potassium directly stimulates aldosterone production, whereas a decrease in serum potassium has the opposite effect. Because of its early point of action in the steroidogenic pathway, ACTH also increases secretion of aldosterone, although it is much less potent in this regard than in its stimulation of cortisol. The stimulatory effects of potassium and ACTH on aldosterone secretion can be overcome by angiotensin II stimulation.
Primary hyperaldosteronism is characterized by mineralocorticoid hypersecretion which promotes a positive sodium balance and hypokalemia. About 80% of patients with primary hyperaldosteronism have serum potassium levels of 3.5 mEq/L or less. Causes of secondary hyperaldosteronism are related to increased renin secretion. These include renal artery stenosis, congestive heart failure and renal salt-wasting.


72. A term neonate is noted to have ambiguous female genitalia. This infant is at risk for which of the following potentially life-threatening problems?

a. Cardiomyopathy with congestive heart failure
b. Sodium wasting nephropathy with hypovolemia
c. Respiratory failure from surfactant deficiency
d. Spontaneous hemorrhage from thrombocytopenia
e. Pulmonary embolus from a hypercoaguable state
Answer: b

Enzymatic defects in the steroidogenic pathway produce a syndrome known as congenital adrenal hyperplasia. This syndrome presents predominantly in the neonatal period with sexual ambiguity. These enzymatic defects result in a lowered cortisol secretion. The specific enzyme defects present determine the clinical form of the syndromes. These include a 21-hydroxylase deficiency, an 11b-hydroxylase deficiency, and a 17-hydroxylase-deficiency. The 21-hydroxylase deficiency and the 11b-hydroxylase deficiency result in excess androgen production in utero and result in masculinization with ambiguous genitalia in the female newborn. Masculinizing effects in the male may not be detected until precocious puberty becomes obvious. About 40% of patients with 21-hydroxylase deficiency, the most common form, have salt-wasting or sodium loss by urine. Hypovolemic shock can result. Cardiomyopathy, respiratory failure, thrombocytopenia and pulmonary emboli are not associated with this syndrome.


73. Which of the following adrenal lesions can be treated definitively by medical means?

a. Benign functional adrenocortical adenoma
b. Adrenocortical carcinoma
c. Congenital adrenal hyperplasia
d. Cushing disease
e. Pheochromocytoma
Answer: c

The treatment of adrenal tumors is primarily surgical removal. Although pharmaceutical agents are useful in preparing the patient for surgery or in palliating the patient with recurrent adrenal carcinoma, no agents render definitive therapy for adrenal tumors. Congenital adrenal hyperplasia stands alone among the primary, hyperfunctioning adrenal syndromes that are amenable to medical therapy for definitive treatment. Functioning benign lesions of the adrenal cortex that are not ACTH dependent, such as adenomas or macronodular hyperplasia, respond to metyrapone and aminoglutethimide, which are inhibitors of enzymes in the adrenal steroidogenic pathway. Both agents can effect a decrease in the production of cortisol when there is no increase in ACTH secondary to feedback stimulation. These drugs are not satisfactory long-term agents because of their high incidence of drug reactions, patient noncompliance, and continued growth of the lesions. They may be useful in patients whose surgery must be delayed. Although malignant, functioning adrenocortical lesions should be debulked whenever possible. Several chemotherapy agents offer adjunct therapy. The most noteworthy is mitotane (o,p,-DDD). This is a cytolytic agent that has a 30% to 70% response rate in terms of decreasing steroid output. Unfortunately, patient survival is not affected. As mentioned, nonoperative treatment with cortisone acetate and possibly fludrocortisone is definitive therapy for congenital adrenal hyperplasia. Cushing disease is best treated by transsphenoidal resection of the pituitary adenoma. Pheochromoctyoma requires definitive surgical resection although the preoperative pharmacologic preparation with catecholamine blockade is required.

74. Which of the following statements regarding the pituitary gland are true?

a. ADH is a product of the neurohypophysis
b. The preferred surgical approach to the pituitary gland is via the sphenoid sinus
c. Growth hormone, ACTH, LH, FSH and serotonin are products of the adenohypophysis
d. The adenohypophysis is regulated by neurotransmitters released by the supraoptic hypophyseal tract
Answer: a, b

The anterior pituitary gland is the adenohypophysis which constitutes 80% of the gland. The posterior pituitary, the neurohypophysis, constitutes the remainder and should be considered virtually an extension of the hypothalamus of the brain. The pituitary resides within bony confines of the sella turcica (Turkish saddle) and is bordered laterally by the cavernous sinuses (venous), inferiorly and anteriorly by the sphenoid sinus (air), posteriorly by the dorsum sella and superiorly by the membranous diaphragma sella. The cavernous sinuses each contain the siphon region of the internal carotid artery and portions of the cranial nerves III, IV, V and VI all within the venous plexus. The optic chiasm lies immediately above the diaphragma sella. Directly below the anterior and inferior portions of the sella is the aerated sphenoid sinus. This is sufficiently large in 97% of the patients to allow a transnasal, transsphenoidal surgical approach to the pituitary.
The adenohypophysis is regulated by a portal venous system between the median eminence of the hypothalamus and the adenohypophysis itself. This system involves a transport of (1) thyrotropin-releasing hormone (TRH), to stimulate the secretion of the thyroid-stimulating hormone (TSH); (2) corticotropin-releasing hormone, to stimulate adrenocorticotropic hormone (ACTH); (3) growth hormone-releasing hormone, to stimulate secretion of growth hormone (GH); (4) gonadotropin-releasing hormone, to stimulate luteinizing hormone (LH) and follicle-stimulating hormone (FSH); and (5) prolactin-inhibitory factor (dopamine), to inhibit prolactin. The neurohypophysis is regulated by means of direct transport of hormones through nerve fibers from the supraoptic and paraventricular nuclei in the hypothalamus. The neurohypophysis is a virtual extension of the hypothalamus. Products of the neurohypophysis are antidiuretic hormone (ADH; vasopressin) and oxytocin. The pituitary gland is not known to release serotonin.


75. Which of the following statement(s) is/are true with respect to growth hormone secreting pituitary adenomas?

a. Fewer than 50% of patients will have growth hormone levels over 10 ng/mL
b. Oral glucose administration suppresses growth hormone levels in patients with acromegaly
c. Over 80% of growth hormone-secreting microadenomas can be cured with transphenoidal resection
d. Preoperative treatment of macroadenomas with a somatostatin analogue may improve postoperative remission rates
Answer: c, d

The endocrine diagnosis of acromegaly rests largely on serum growth hormone (GH) levels, because 90% of patients will have levels over 10ng/mL. When acromegaly is apparent but consistently elevated growth hormone levels are not obtained, the glucose suppression test is the most useful diagnostic procedure. In normal patients, 1 to 2 hours after the oral administration of 100 g of glucose, the growth hormone level falls well below 5 ng/mL. This suppression is not seen with GH-secreting adenomas, and often a paradoxical rise in GH is observed.
The goals of treatment are to lower the circulating growth hormone or somatomedin C levels to within a normal range and to reduce the size of the mass lesion causing compression-related symptoms. When a microadenoma is removed transsphenoidally, endocrine remission may be expected in 80% to 88% of cases. When a macroadenoma is resected, postoperative remission is reported in 30% to 68% of cases. The rate of remission is inversely related to preoperative GH levels and tumor size. Preoperative treatment of macroadenomas with a somatostatin analogue may improve postoperative remission rates.


76. A 30-year-old woman presents with amenorrhea, headache and bitemporal hemianopsia. Appropriate diagnostic tests include:

a. Cerebral angiography
b. Serum prolactin levels
c. Magnetic resonance imaging of the brain
d. Abdominal and pelvic CT scan
Answer: b, c

Patients with pituitary lesions present symptoms and signs related to a mass effect on the pituitary and its surrounding structures, to hypersecretion of the hormones by the lesion itself, or to a combination of both. As mass lesions in the pituitary enlarge, they encounter the various contents of the cavernous sinuses, including the third, fourth, sixth and first two divisions of the fifth cranial nerves, as well as the internal carotid artery. The growth of a tumor in the relatively unrestricted upward direction is much more common and often results in compression of the optic chiasm with the resultant loss of vision, typically a bitemporal hemianopsia. Prolactin-secreting pituitary adenomas often present with endocrine symptoms including amenorrhea and galactorrhea in women. In men, the loss of libido, infertility and visual loss are typical. Magnetic resonance imaging (MRI) has evolved as the first choice for diagnostic imaging and is often the only tool needed to reach a therapeutic decision with regard to pituitary adenomas. With intravenous infusion of a paramagnetic substance such as gadolinium, MRI demonstrates intrasellar tumors as small as 5 mm. In addition, the extent of suprasellar and sphenoid sinus extension, as well as lateral extension into the cavernous sinuses, is demonstrable. Cysts and hemorrhage can be differentiated, as can blood flowing within an aneurysm. CT scanning has a place in pituitary imaging if MRI scanning is unavailable. Plain skull X-rays are not needed generally. Cerebral angiography is performed only if an aneurysm is suspected or if a lesion is so large that occlusion or compression of the internal carotid artery is in question. For this patient, the symptoms clearly point to a central nervous system, pituitary etiology rather than abdominal end organ failure with regard to the amenorrhea.


77. Which of the following condition(s) is/are associated with hyperprolactinemia?

a. Chronic renal failure
b. Exogenous estrogen administration
c. Diabetes mellitus
d. Cirrhosis
Answer: a, b, d

Elevated serum prolactin levels do not always indicate the presence of a pituitary tumor. Important alternative causes are chronic renal failure, hypothyroidism, various drugs including phenothiazines, tricyclic antidepressants, exogenous estrogen, opiates, reserpine, verapamil and others. In addition, hepatic disease, pregnancy and a variety of pituitary and hypothalamic lesions cause hyperprolactinemia. If the prolactin level is over 150 ng/ml, a pituitary tumor is almost invariably the cause, but often microadenomas produce prolactin levels of less than 100 ng/ml. The size of pituitary tumors has been shown to relate to the degree of prolactin elevation, which may reach into the thousands of nanograms per milliliter. There are no reliable provocative tests to differentiate prolactinomas from other causes of hyperprolactinemias, so the diagnosis relies on ruling out other causes and imaging of the adenoma.


78. Pituitary adenomas are best classified according to functional hormone output. This information may be derived from which of the following?

a. Hematoxylin and eosin staining
b. Immunohistochemical staining of pituitary tissue
c. In situ hybridization studies
d. Selective venous sampling from the inferior petrosal sinuses
Answer: b, c, d

Pituitary adenomas have been classified historically as acidophilic, basophilic and chromophobic. Adenomas may show a variable staining pattern with conventional hematoxylin and eosin dyes, so it is difficult to classify adenomas based on these stains. Immunohistochemistry, ultrastructural studies and in situ hybridization analyses for specific hormones are the most reliable methods of classifying pituitary adenomas today. Immunohistochemical staining of pituitary adenomas with specific antibodies has reliably classified adenomas using highly purified polyclonal and monoclonal antibodies against prolactin, GH, ACTH, FSH-b LH-b, and TSH-b. Many studies with these antibodies have revealed that some pituitary tumors are composed of several cell types, which produce various hormones. Some adenomas may not store specific hormones, so immunohistochemical staining may be weak or absent. The mRNA is usually present in the cytoplasm of adenomas. Localization of mRNA for specific protein hormones is becoming more widely used in the classification of pituitary adenomas. Null-cell adenomas constitute up to 25% of pituitary neoplasms. Selective venous sampling from the inferior petrosal sinus via transfemoral catherization is an effective method to compare venous effluent from the pituitary to systemic levels for a specific hormone. In addition, this technique may demonstrate laterality. This latter issue is potentially important as certain small adenomas may not be discernible from the gross appearance at surgery.


79. Which of the following statements is true with respect to Cushing Disease?

a. Pituitary microadenomas are often small and deep within the gland itself
b. The treatment of choice for hypercortisolism due to a pituitary adenoma in women of childbearing age is transsphenoidal total hypophysectomy
c. Patients who fail to remit with both surgery and radiation to the pituitary require either medical or surgical adrenalectomy
d. The long-term recurrence rate after resection of an ACTH-producing pituitary microadenoma is approximately 40%
Answer: a, c

Pituitary microadenomas secreting ACTH may be very small and are often located deep within the gland itself. If the tumor is not evident on opening the dura, incisions must be made into the gland and internal exploration carried out. If no tumor is identified, then a decision must be made as to whether to resect all or a portion of the gland. If the endocrine evidence is convincing for pituitary origin and the patient has no desire to have children, then total hypophysectomy is warranted. If the petrosal sinus sampling clearly indicates laterality then appropriate hemiresection of the gland may be done.
About 75% of patients have microadenomas as the source of ACTH secretion. The postoperative remission rate in these patients is 88% to 96%, and the long-term recurrence rate appears to be no more than 5%. 10% to 20% of patients who undergo exploration have macroadenomas, and the postoperative remission rates in these patients have been reported to be from 33% to 61%. Most of these patients require postoperative radiation therapy. Patients who fail both surgery and radiation require either surgical adrenalectomy or medical suppression of adrenal function.


80. A 45 year-old woman presents for evaluation of hypertension, recent onset obesity, hirsutism and depression. Cerebral MRI does not show a pituitary lesion. Evaluation may include determination of which of the following?

a. AM serum cortisol levels after low dose dexamethasone suppression
b. Simultaneous serum ACTH measurement in peripheral and inferior petrosal sinus sites
c. Chest and abdominal CT scan
d. Urinary free cortisol excretion
Answer: b, c, d

The findings of Cushing Syndrome often include central obesity, hypertension, hirsutism, fatigue, easy bruisability, stria, moon-like facies, dorsal fat pad, and often depression or other mental changes. Less common abnormalities include headache, osteoporosis, diabetes mellitus, galactorrhea, peripheral edema and amenorrhea. Often, a patient presents without the classic cushingoid appearance and complains only of severe fatigue or depression. The cause of hypercortisolism is an ACTH-secreting pituitary adenoma (Cushing disease) in up to 80% of cases, with remainder due either to an adrenocortical tumor or to an ectopic neoplasm secreting ACTH or corticotropin-releasing factor. Pituitary-dependent hypercortisolism is much more common in women(80%) and an ectopic etiology more common in men.
Up to 60% of patients with pituitary etiologies have nondiagnostic imaging studies, therefore, the diagnosis often relies completely on endocrine testing. Multiple measurements of cortisol and ACTH to evaluate the diurnal pattern are important but often misleading. They are mainly of value when clearly elevated. The determination of urinary free cortisol excretion over 24 hours is an extremely important measurement. If the overnight dexamethasone screening test yields an 8 AM serum cortisol level of less than 5 ug/dl, then hypercortisolism is rarely present. Generally, patients with a pituitary etiology of hypercortisolism do not show suppression with the low-dose dexamethasone test, but do with the higher dose test. Patients with adrenal or ectopic etiologies do not experience suppression with either dose. Chest and abdominal CT scans are appropriate to look for adrenal or lung tumors. The most specific test when the MRI is negative and evidence implicates the pituitary, is simultaneous measurement of ACTH levels in both inferior petrosal sinuses and a concurrent determination of the peripheral ACTH level. This approach produces specific information about the existence of an ACTH-secreting pituitary tumor and even the laterality of the tumor.


81. The most common mass lesion in the sella turcica is which of the following?

a. Craniopharyngioma
b. Aneurysm
c. Benign pituitary cyst
d. Pituitary adenoma
Answer: d

Pituitary adenomas are the most common mass lesions in the sella turcica or parasellar region. They constitute 8% to 10% of all brain tumors. Occasionally, they are cystic and may be confused with other lesions. Craniopharyngiomas are the next most common tumor, although these are more often suprasellar in location. These are more common in children, but up to one third occur in adults. They are usually cystic and are calcified in 70% of children and 40% of adults. More rare lesions include meningiomas, germinomas, metastatic malignancies from lung and breast primaries, gliomas, dermoids and benign epidermoid. Rathke cysts, aneurysms, and a variety of inflammatory and granulomatous processes.


82. Pharmacologic treatment of growth hormone (GH) excess secondary to a pituitary adenoma may include the use of which of the following?

a. Bromocriptine
b. Vasopressin
c. Octeotide
d. Prednisone
Answer: a, c

Bromocriptine, a dopamine receptor agonist, has been demonstrated to lower GH levels in 71% of 126 acromegalic patients. A clinical response was achieved in up to 95% of acromegalic patients, and reduced somatomedin C levels were found in some patients with persistently elevated GH levels. Bromocriptine does not appear to be an effective primary treatment for acromegaly, but may help to control GH and somatomedin C levels as an adjuvant therapy. A somatostatin analogue, octeotide, has recently been used to treat acromegaly and has been demonstrated to significantly reduce GH and somatomedin C levels in most patients and normalize values in 50%. This treatment provides only minimal tumor shrinkage, and GH levels rise again immediately following cessation of the drug. This drug may prove to be useful as a preoperative treatment or in surgical failures. Vasopressin and prednisone have no role in the treatment of acromegaly.

4 comments:

  1. I was diagnosed with hypothyroidism two years ago and my pharmacist prescribed desiccated bovine supplements. After few weeks I felt so much better!

    ReplyDelete
  2. My Mum who was experiencing weight gain, hair loss and acne, bouts of chest and abdominal pain and several months of unrelenting headaches that no medicine could relieve, In fact, it got worse. saw various doctors they decided that the lab levels indicated nothing much at all, until we met a Doc. who did CT scan, and she was diagnosed with adrenocortical carcinoma. So after one year, A surgeon advice was to remove her tumor, along with her left kidney. she needed to see someone with extensive experience in treating adrenal gland tumors for the next phase of her treatment. Knowing she needed to get an answer once and for all, we looked up on the internet and saw someone who testified about Aryan herbal medicine which she took and she had no more symptoms since then. You can also contact him if interested at
    draryan500@gmail.com

    ReplyDelete
  3. My husband was diagnosed with early onset Parkinson's disease at 57.his symptoms were shuffling of feet,slurred speech, low volume speech, degradation of hand writing, horrible driving skills, right arm held at 45 degree angle, things were tough for me, but now he finally free from the disease with the help of total cure ultimate health home, he now walks properly and all symptoms has reversed, he had trouble with balance especially at night, getting into the shower and exiting it is difficult,getting into bed is also another thing he finds impossible.we had to find a better solution for his condition which has really helped him a lot,the biggest helped we had was ultimatehealthhome they walked us through the proper steps,am highly recommended this ultimatehealthhome@gmail.com to anyone who needs help.

    ReplyDelete

Related Posts Plugin for WordPress, Blogger...
Related Posts Plugin for WordPress, Blogger...